You are on page 1of 63

ERRNVPHGLFRVRUJ

Manual CTO
de Medicina y Cirugía

3 .ª edición

Urología

Revisores
Felipe de Jesús Medina Toscano
Karla Leonor Sierra Guerra
Pablo Conde Caturla
Claudio Daniel Rojas Gutiérrez
(revisor preguntas ingles)
Carlos Cuauhtémoc Plazola Stephens
(revisor preguntas español)

Autores
Luis Cabeza Osorio
Carlos de la Puente Bujidos
Felipe de Jesús Medina Toscano
Carlos Eduardo Sandoval Castro

Grupo CTO
• • Editorial
NOTA

La medicina es una ciencia sometida a un cambio constante. A medida que la investigación y la experiencia
clínica amplían nuestros conocimientos, son necesarios cambios en los tratamientos y la farmacoterapia.
Los editores de esta obra han contrastado sus resultados con fuentes consideradas de confianza,
en un esfuerzo por proporcionar información completa y general, de acuerdo con los criterios aceptados
en el momento de la publicación. Sin embargo, debido a la posibilidad de que existan errores humanos
o se produzcan cambios en las ciencias médicas, ni los editores ni cualquier otra fuente implicada
en la preparación o la publicación de esta obra garantizan que la información contenida en la misma sea
exacta y completa en todos los aspectos, ni son responsables de los errores u omisiones ni de los resultados
derivados del empleo de dicha información. Por ello, se recomienda a los lectores que contrasten dicha
información con otras fuentes. Por ejemplo, y en particular, se aconseja revisar el prospecto informativo
que acompaña a cada medicamento que deseen administrar, para asegurarse de que la información
contenida en este libro es correcta y de que no se han producido modificaciones en la dosis recomendada
o en las contraindicaciones para la administración. Esta recomendación resulta de particular importancia
en relación con fármacos nuevos o de uso poco frecuente. Los lectores también deben consultar
a su propio laboratorio para conocer los valores normales.

No está permitida la reproducción total o parcial de este libro, su tratamiento informático, la transmisión
de ningún otro formato o por cualquier medio, ya sea electrónico, mecánico, por fotocopia, por registro
y otros medios, sin el permiso previo de los titulares del copyright.

© CTO EDITORIAL, S.L. 2017

Diseño y maquetación: CTO Editorial

C/ Albarracín, 34; 28037 Madrid


Tfno.: (0034) 91 782 43 30 - Fax: (0034) 91 782 43 43
E-mail: ctoeditorial@ctomedicina.com
Página Web: www.grupocto.es

ISBN Urología: 978-84-17095-98-7


ISBN Obra completa: 978-84-17095-72-7
Depósito legal: M-29877-2017
ERRNVPHGLFRVRUJ
Urología

Manual CTO
de Medicina y Cirugía

3.ª edición

GrupoCTO
. . Editorial
Q)
u
·-
-0
e
,_ ca
..,_
e
C')
04. Tumores renales ...............................................................................
4.1.

4.2.
Carcinoma de células renales
(adenocarcinoma renal, hipernefroma) _ _ _ 18
Otros tumores
18

20
e....
::::::,
05. Hiperplasia y carcinoma prostático ..... 22
5.1. Hiperplasia prostática benigna. _ _ _ _ _ 22
5.2. Carcinoma prostático 24
01. Semiología urológica y definiciones. 1
1.1. Definiciones _ _ __ _ __ _ _ __
1.2. Diagnóstico diferencial
06. Carcinomas del tracto urinario........................ 32
de la hematuria macroscópica _ _ _ __ 2 6.1. Carcinoma vesical _ _ _ _ _ _ _ _ _ 32
6.2. Tumores del tracto urinario superior 34

02. Infecciones del tracto urinario.


C.1st1t1s
.. . . . 1
1nterst1c1a ........................................................................... 3 07. Tumores testiculares .............................................................. 37
2.1. Patogénesis y etiología _ __ _ _ _ __ 3 7.1. Etiología y epidemiología 37
2.2. Diagnóstico _ __ _ _ _ _ _ _ __ 4 7.2. Anatomía patológica 37
2.3. Diferentes ITU y su tratamiento _ _ _ __ 4 7.3. Clínica 38
2.4. Tuberculosis genitourinaria _ __ _ __ 6 7.4. Diagnóstico 38
2.5. Cistitis intersticial _ _ _ _ _ _ _ __ 7 7.5. Diagnóstico diferencial 39
7.6. Tratamiento 40

03. Urolitiasis........................................................................................................... 10
3.1. Epidemiología _ _ _ _ _ _ _ _ _ _ 10
08. Trasplante renal .................................................................................. 43
3.2. Manifestaciones clínicas y su manejo agudo ....... 10 8.1. Indicaciones _ _ _ _ _ _ _ _ _ _ _ 43
3.3. Evaluación y tratamiento de la litiasis renaL............. 12 8.2. Complicaciones 43

ERRNVPHGLFRVRUJ
,
Urología i I n d iC e

09. Uropatía obstructiva ................................................................ 45 11. Traumatismos del aparato


9.1. Características 45 genitourinario.. ........... . ......................................................................... 50
9.2. Clínica 45 11.1 . Lesiones del riñón 50
9.3. Diagnóstico 45 11.2. Lesiones del uréter 51
9.4. Tratamiento 46 11.3. Lesiones de la vejiga 51
11.4. Lesiones de la uretra 52
11.5. Lesiones del pene 52
10. Disfunción eréctil................ ............................................................ 47 11.6. Lesiones de los testículos 52
10.1 . Introducción 47
10.2. Prevalencia 47
10.3. Etiología 47 Bibliografía 57
10.4. Factores de riesgo 47
10.5. Diagnóstico 47
10.6. Tratamiento 47

ERRNVPHGLFRVRUJ
_Urología

Semiología urológica y definiciones

Este tema puede ayudar a tes quirúrgicos previos) y la segunda, el uréter ectópico (que es
obtener una visión general de la causa más frecuente en niñas).
la materia y a asociar algunos De esfuerzo: se desencadena con el aumento de presión abdo-
conceptos básicos, pero no se minal (al reír, toser, cargar con peso). Generalmente se produce
debe emplear en él demasiado por un déficit de soporte de la musculatura perineal (por ello es
tiempo.
recomendable revisar los antecedentes obstétricos, pacientes
obesas, pacientes añosas, etcétera).
De urgencia: el paciente siente ganas de orinar, pero no le da
1.1 . Definiciones tiempo a llegar al baño (provocadas por contracciones involun-
tarias del músculo detrusor).
Mixta: generalmente es una combinación de las dos anteriores.
Hematuria microscópica: presencia de más de 5 hematíes por Paradójica: escape de orina debido a la sobredistensión vesi-
campo. La causa más frecuente en ambos sexos es la litiasis. La causa cal. El ejemplo característico es el paciente prostático con re-
mas común en varones mayores de 50 años es la hiperplasia benig- tención urinaria. La presión intravesical supera la presión de
na de próstata. cierre del esfínter uretral, produciéndose un escape de orina
Hematuria macroscópica: orina de aspecto rojizo a simple vista paradójico (no puede orinar y, sin embargo, se le escapa la
debido a la presencia de más de 100 hematíes por campo. En los orina).
pacientes fumadores, en ausencia de otros síntomas, se debe sospe-
char tumor maligno de vías urinarias.
Piuria: presencia de más de 1O leucocitos por campo. Altamente
Los cilindros hemáticos aparecen en las glomeru-
inespecífica, pero en presencia de síntomas urinarios, hay que sos- lonefritis que producen síndrome nefrítico, como
pechar infección. en la postinfecciosa.
Síndrome miccional: presencia de polaquiuria (aumento en la fre-
cuencia miccional), urgencia miccional (necesidad imperiosa e irre-
frenable de orinar) y disuria (molestias urinarias inespecíficas referi- Enuresis: pérdidas de orina exclusivamente durante el sueño. Si el
das como ardor, escozor, etcétera). niño es mayor de 6 años, debe ser estudiado.
Incontinencia urinaria: pérdidas involuntarias de orina (Tabla 1). Crisis renoureteral: dolor lumbar frecuentemente irradiado a ge-
Existen cuatro tipos principales: nitales, de carácter agudo, cuya intensidad no se modifica por los
Continua: de día y de noche, en todas las posiciones. La causa cambios posturales, y que se suele acompañar de náuseas, vómitos
más frecuente es la fístula urinaria (en pacientes con anteceden- y malestar general. Es muy poco frecuente que sea bilateral.

Síntomas Incontinencia de urgencia Incontinencia de esfuerzo

Urgencia (deseo repentino de orinar) Sí No

Aumento de la frecuencia miccional Sí No

Capacidad de llegar al baño después de sentir el deseo de orinar No Sí


Despertarse para ir al baño durante la noche Sí Generalmente no
Escape durante la actividad física No Sí
Cantidad de orina escapada en cada episodio de incontinencia Abundante, si se produce Generalmente escasa
Tabla 1. Diagnóstico diferencial de la clínica de incontinencia urinaria femenina

ERRNVPHGLFRVRUJ
Manual CTO de Medicina y Cirugía, 3.ª edición

1.2. Diagnóstico diferencial Hematuria con coágulos: indica un problema urológico. La causa
más frecuente en mujeres es la cistitis hemorrágica, aunque la pri-
de la hematuria macroscópica mera causa a descartar es una neoplasia urotelial, máxime en el pa-
ciente fumador.
Según el momento de aparición: Hematuria sin coágulos: Generalmente asociado a nefropatía,
Inicial: sangrado uretral o prostático. puede ir acompañada de cierto grado de proteinuria, así como de
Final: sangrado del cuello vesical. cilindros eritrocitarios o de hematíes dismórficos en el sedimento
Total: vesical o del tracto urinario alto. Se debe recordar que un urinario, rara vez etiología por patología urológica ..
sangrado importante de cualquier parte del aparato genitouri-
nario puede provocar hematuria total.

" La hematuria con coágulos indica un problema urológico.


Ideas clave
" La causa más habitual de hematuria es la cistitis hemorrágica,
" La causa más frecuente de hematuria microscópica es la litiasis pero lo primero a descartar es el tumor urotelial.
(en la población general, en ambos sexos).
" Los hematíes dismórficos en el sedimento orientan a nefropatía
" La causa más común de hematuria microscópica en varones de de origen glomerular.
más de 50 años es la hiperplasia benigna de próstata.

A 63-year-old woman presents to the emergency department


Case Study complaining of frequent urine leakage. She says that these leaks
are more prominent when coughing, laughing or sneezing. She
A 7-year-old boy is brought to our clinic with a history of noctu- also claims that at times, she is una ble to reach the bathroom.
ria, 5-6 times a week. Regarding this clinical case, which of the
following is false? 1) Symptoms are consistent with emergency urinary incontinence
and should be treated with anticholinergic agents.
1) lt should be considered monosymptomatic nocturnal enuresis 2) Symptoms are consistent with emergency urinary incontinence
providing the patient does not have daytime symptoms. and the patient should initially perform pelvic floor exercises.
2) The usual clinical course tends to resolve spontaneously. 3) lf these exercises result ineffective, a suburethral mesh place-
3) Usually, these kids continue presenting urinary sphincter pro- ment is indicated.
blems until adulthood. 4) In this case, pelvic floor muscle training will not be effective.
4) In 60% of cases, family history may be encountered.

01 · Semiología urológica y definiciones


ERRNVPHGLFRVRUJ

Infecciones del tracto urinario.


Cistitis intersticial

Este tema es el más importante de esta asignatura. Hay que tener en cuenta la tuberculosis
genitourinaria, la cistitis intersticial, pielonefritis e infecciones urinarias durante el embarazo
ENARM suelen ser temas preguntados y rentables al momento de ser evaluados

La infección del tracto urinario (ITU) puede clasificarse de varias formas. Otro dato que apoya la importancia de la vía ascendente es la frecuencia
Se puede hacer una división anatómica entre las ITU altas (infecciones de infección tras el cateterismo uretral, que es del 1% en los pacientes
renales) y las ITU bajas (cistouretritis, prostatitis). Asimismo, la clasifica- ambulantes, y en tres o cuatro días alcanza a casi la totalidad de los pa-
ción puede basarse en la asociación o no de complicaciones. Una ITU cientes sondados con sistemas de drenaje abiertos. En los pacientes hos-
no complicada es un cuadro clínico caracterizado por la presencia de pitalizados, el riesgo de infección alcanza un 5% por cada día de sondaje,
escozor miccional, urgencia y frecuencia, acompañado o no por hema- incluso con sistemas cerrados.
turia terminal, dolor hipogástrico, y más raramente, febrícula. Dentro de
este grupo se podrían incluir las pielonefritis no complicadas, que se Una vez que las bacterias han alcanzado el tracto urinario, tres factores
presentan como cuadros febriles con hipersensibilidad en fosa lumbar, determinan el desarrollo de la infección:
fieb re, náuseas o vómitos, y sin los factores que convierten la ITU en La virulencia del microorganismo.
"complicada", como son: presencia de catéteres, uropatía obstructiva, El tamaño del inóculo.
reflujo vesicoureteral, anomalías anatómicas, insuficiencia renal o tras- Los mecanismos de defensa del huésped.
plante renal. La ITU en el varón debe considerarse esencialmente "com-
plicada" de entrada. La mayoría de las infecciones en la comunidad están producidas por gér-
menes gramnegativos, principalmente f. co/i, responsable del 85% y, en
La reaparición de una infección tras el tratamiento puede deberse a menor proporción, Proteus, Klebsiella o Pseudomonas. Entre los gramposi-
reinfección o recidiva. El primer término expresa la infección nueva por tivos, únicamente el Staphylococcus saprophyticus tiene relevancia, pro-
un germen distinto al inicial, mientras que recidiva indica infección por duciendo el 10-15% de las ITU en mujeres jóvenes (segundo germen más
el mismo germen. Esta última es mucho más infrecuente que la rein- frecuente en esta población).
fección y puede estar ocasionada por litiasis infectiva, prostatitis cró-
nica, fístulas vaginales o intestinales, divertículos vesicales infectados, En las infecciones nosocomiales, los gérmenes gramnegativos conti-
cuerpos extraños, necrosis papilar infectada y otras causas que generan núan siendo los más frecuentes. Si bien f. co/i es el más habitual, su
un reservorio de microorganismos que difícilmente se eliminan con el frecuencia desciende hasta el 50% y adquieren mayor importancia Pro-
antibiótico. teus, Klebsiella, Pseudomonas, Enterobactery Serratia. El 25% restante está
ocasionado por gérmenes grampositivos como estreptococos y estafi-
lococos. Candida.
2.1. Patogénesis y etiología

Existen tres posibles vías por las que los microorganismos pueden alcan- Uropatogenos que requieren cultivos especiales:
zar el tracto urinario: hematógena, linfática y ascendente. La vía linfática Anaerobios, Mycobacterias, Chlamydia.
carece de importancia real. La diseminación hematógena tampoco es
frecuente. La más común es la ascendente iniciada en la uretra. Probable-
mente por esta razón es mucho más habitual la ITU en mujeres, dado que La afectación del tracto urinario superior parece también producirse por
su uretra es muy corta y ancha, y por ello favorece el paso de microorga- ascenso de los gérmenes a lo largo del uréter. La diferenciación, aunque
nismos hacia niveles más altos del TGU. poco específica, se debe basar en los hallazgos clínicos (fiebre, dolor lum-

ERRNVPHGLFRVRUJ
Manual CTO de Medicina y Cirugía, 3.ª edición

bar, escalofríos) y analítica elemental (leucocitosis, velocidad de sedimen- Pacientes con bacteriuria asintomática e infecciones recurrentes no es re-
tación alta). comendable tratar si no hay síntomas, se debe de buscar factores predis-
ponentes. En varones se debe de sospechar de prostatitis crónica como
primer opción.
2.2. Diagnóstico
Mujeres embarazadas se correlaciona con episodios de pielonefritis, así
como parto pretermino, el screening y tratamiento está recomendado
El diagnóstico de ITU, además de la clínica, se define por el cultivo de en estos pacientes.
orina. Dado que es frecuente el crecimiento de bacterias que han con-
taminado las muestras, se utiliza un criterio estadístico sobre la base del Situaciones especiales: diabetes mellitus controlada no se recomienda
recuento de colonias del urocultivo, considerando como significativo clá- tratamiento, mal controlada se puede considerar tratamiento por au-
sicamente el crecimiento de más de 1oscolonias por mililitro. mento de incidencia de IVU complicada, vejiga neurogenica, derivacio-
nes urinarias como conductos ileales, lesión espinal, catéter urinario o
En determinadas circunstancias, recuentos de colonias menores pue- nefrostomias frecuentemente están colonizados, no requieren trata-
den ser suficientes: recuentos de 103 UFC/ml en mujeres sintomáticas, miento.
más de 104 en pielonefritis clínicas o en varones, y más de 102 en mues-
tras de cateterismos limpios o cualquier recuento, si se recoge median- ITU baja en mujeres
te punción-aspiración supra púbica. Cifras mayores de 1os UFC/ml pue-
den igualmente reflejar contaminación, principalmente si crecen dos o Puede realizarse un tratamiento convencional de siete días o bien un cur-
más especies. so corto en monodosis o en régimen de tres días. La ventaja de estos es
el menor coste económico y la menor incidencia de efectos adversos. Su
En el adulto, la presencia de pi u ria (más de 1Oleucocitos/mm3) se relacio- desventaja es la mayor incidencia de recurrencias tempranas, al no afee-
na estrechamente con la ITU en presencia de síntomas, no así en el niño, tar apenas a los reservorios vaginal e intestinal de uropatógenos. Aun con
en el que puede acompañar a los cuadros febriles. todo, por las ventajas mencionadas, la paut a preferida actualmente es el
tratamiento de tres días.

2.3. Diferentes ITU y su tratamiento En una mujer no embarazada y sana se recomienda esquema corto de 3
dias, los antibióticos de elección deben ser el cotrimazol como primera
elección (en decremento su uso en nuestro medio por el elevado índice
En el tratamiento de la ITU lógicamente es fundamental el empleo de de resistencias), la nitrofurantoína de segunda elección, otros:la fosfomi-
antimicrobianos. El número empleado de estos es elevado y las pautas cina, las fluoroquinolonas, las cefalosporinas y otros beta-lactámicos de-
de tratamiento muy variables. A continuación, se repasarán las opciones ben reservarse para los cuadros complicados, con mala respuesta previa
terapéuticas según el tipo de ITU a la que uno se enfrente. o graves.

Bacteriuria asintomática En mujeres embarazadas se recomiendan las pautas largas de tratamien-


to (siete días), evitando el uso de sulfamidas al final del embarazo por
Definida como bacteriuria significativa (1 os UFC/ml) en, al menos, dos el riesgo incrementado de kernicterus, y el empleo de quinolonas por el
urocultivos con el mismo germen, tomados con una semana de diferen- daño producido sobre el cartílago de crecimiento fetal. Tampoco se em-
cia en ausencia de síntomas. La bacteriuria asintomática no debe tratarse plearán pautas cortas en caso de sospecha de pielonefritis, presencia de
salvo en los casos en los que conlleva un riesgo de infección clínica o cálculos o anomalías de la vía urinaria, o bien infecciones previas por mi-
daño orgánico, como ocurre en niños menores de 5 años, tengan o no croorganismos resistentes a los antibióticos.
patología urológica asociada. Asimismo, debe ser tratada en el embara-
zo, en pacientes inmunodeprimidos, como profilaxis previa a una cirugía Tratamiento de primera elección Amoxicilina durante 7 días o Nitrofuran-
urológica (Tabla 1). toína durante 7 días.

ITU recurrente
Menores de 5 años
Embarazadas Aparición de tres o más episodios en 12 meses. Se puede realizar pro-
lnmunodeprimidos filaxis con cotrimoxazol o una fluoroquinolona (en función de la sensi-
Previamente a cirugía urológica bilidad del germen aislado en el último episodio) en dosis única, días
Tabla 1. Bacteriuria asintomática: indicaciones de tratamiento alternos, durante seis meses. Si tras la retirada se presentaran nuevas
recurrencias, puede reinstaurarse el tratamiento durante periodos más
Pacientes sin factores de riesgo la bacteriuria asintomática no representa prolongados (1-2 años). Es aconsejable la ingesta abundante de agua y
riesgo de daño renal, no requiere screening ni tratamiento. realizar micciones frecuentes y cumplir una serie de reglas básicas higié-
nico-dietéticas.

Proteus es intrínsecamente resistente a la nitrofuran- Si los episodios tienen relación con el coito, se puede administrar un
toína, ya que alcaliniza la orina gracias a su ureasa, y comprimido de cotrimoxazol o una quinolona después del mismo. En
este grupo de antibióticos únicamente es útil en me-
mujeres posmenopáusicas, el tratamiento con estrógenos tópicos vagi-
dio ácido.
nales disminuye la frecuencia de infecciones.

02 · Infecciones del tracto urinario .


Cistitis intersticial
ERRNVPHGLFRVRUJ
Urología 1 02
E. coli. Durante la inflamación aguda, los antibióticos penetran adecuada-
Staphylococcus saprophyticus se ha relacionado con mente, pero una vez que esta cede, la penetración es más pobre. Por ello,
ITU en mujeres jóvenes sexualmente activas. se deben utilizar cursos largos de tratamiento (3-4 semanas) para intentar
evitar la persistencia de focos que den pie a una prostatitis crónica. Entre
los antimicrobianos empleados, las fluoroquinolonas son las que mejor
Pielonefritis aguda no complicada difunden al tejido prostático.

El cuadro clínico se caracteriza por dolor en flanco, nausea o vomito, fie-


bre y sensibilidad en ángulos costrodiafragmanticos. Laboratorio: Uriana-
El Antígeno prostático especifico puede elevarse en
lisis y Urocultivo debe ser indicado de rutina, estudio de imagen inicial presencia de prostatitis; y puede tardar hasta 3 meses
USG de tracto urinario superior buscando obstruccion del flujo urinario, en normalizarse posterior al tratamiento.
la Tomografía simple, urograma excretor o Gamagrama renal deben de
considerarse en base a evolución
La prostatitis crónica bacteriana suele presentarse como molestias
En los casos de gravedad leve-moderada, puede plantearse terapéutica perineales o genitales, síntomas irritativos (polaquiuria, tenesmo, es-
oral con cotrimoxazol (en desuso en nuestro medio por el elevado índice cozor) y episodios de ITU recurrentes causados por el mismo orga-
de resistencias), fluoroquinolonas o ~-lactámicos. En pacientes graves u nismo. En el líquido prostático se evidencian más de 1O leucocitos
hospitalizados es preciso tratamiento parenteral, y el espectro de antimi- por campo de gran aumento, y macrófagos que contienen cuerpos
crobianos incluye ampicilina (enterococo), ureidopenicilinas (Pseudomo- ovales grasos.
nas), cefalosporinas de segunda o tercera generación, e incluso amino-
glucósidos. Nunca se emplearán pautas cortas. El tratamiento debe estar guiado por los cultivos, tanto de orina como
de fluido obtenido por masaje prostático, y prolongarse entre 4 y 16 se-
La duración del tratamiento debe ser por 14 días parenteral o vía oral y la manas. El síndrome doloroso pelviano crónico (SDPC) hace referencia a
respuesta clínica se debe de valorar a las 48 a 72 horas. molestias genitourinarias de más de tres meses de evolución con cultivos
habituales negativos. Presenta etiopatogenia multifactorial: infeccioso
En varones siempre se debe de sospechar de factor predisponente, más por gérmenes atípicos (ureaplasma, micoplasma, chlamydias ... ), psico-
frecuentemente obstrucción por crecimiento prostático lógico (estrés), neurológico (similar a fibromialgia y colon irritable). Trata-
miento: antibióticos, antiinflamatorios, antidepresivos, rehabilitación del
ITU en varones suelo pélvico.

Cualquier ITU en varón debe considerarse como complicada inicialmen- Se denomina prostatodinia a un cuadro clínico similar donde predomi-
te, ya que hay que asumir que existe afectación del tejido prostático, re- nan las molestias perineales o genitales con cultivos negativos y menos
nal o que existen problemas concomitantes como obstrucción urinaria, de 1Oleucocitos por campo en el líquido prostático. Su causa es descono-
litiasis o malformaciones urológicas. Por todo ello, el tratamiento debe ser cida y el tratamiento difícil, empleándose actualmente a-bloqueadores o
más prolongado (mínimo una semana), no siendo adecuados los cursos relajantes musculares como terapia inicial.
cortos de tratamiento.
Orquiepididimitis
Prostatitis
En varones menores de 35 años se considera, en el plano teórico, una en-
La infección aguda del tejido prostático se presenta como un cuadro sép- fermedad de transmisión sexual, siendo los agentes más frecuentes Ch/a-
tico con afectación general del paciente, fiebre elevada, síndrome mic- mydia trachomatis y Neisseria gonorrhoeae. El tratamiento sería ceftriaxona
cional, artromialgias y dificultad miccional (Tabla 2). En el examen rectal, (125-250 mg) en dosis única intramuscular o Azitromicina 1 gr VO DU o
la próstata aparece muy dolorosa e inflamada. El germen más habitual es doxiciclina oral durante 7 a 10 dias.

Prostatitis
aguda

Prostatitis
Etiolog1a

E. co/i

E. co/i 1
Clínica

Cuadro séptico

rritativo con
••+

+
+

+/-
Líquido
prostático

> 1Oleucocitos/campo
Cultivo líquido
prostático

Nunca hacer masaje prostático ni sondaje

+
Tratamiento

Cotrimoxazol,
fluoroquinolonas
4 semanas
Cotrimoxazol,
crónica reagudizaciones, sin fluoroquinolonas
bacteriana 6-12 semanas
fiebre ni leucocitosis
Síndrome Desconocida/ Dolor genitourinario Tipo A: inflamatoria: Antibióticos
doloroso Ureap/asma > 3 meses > 1Oleucocitos/campo (gérmenes
pelviano atípicos)
crónico Mycoplasma Tipo B: inflamatoria:
< 10 leucocitos/campo Rehabilitación suelo
pélvico
Antidepresivos
Tabla 2. Diagnóstico diferencial de las prostatitis Clasificacion NIH)

ERRNVPHGLFRVRUJ
Manual CTO de Medicina y Cirugía, 3.ª edición

141%:iHfli Etiología Lesión típica Diagnóstico Tratamiento

Uretritis Neisseria gonorrhoeae Asintomática ( f) Contacto< 5 días Ceftriaxona


gonocócica Exudación uretral matutina ( t) Gram de exudado cervical o espectinomicina
Epididimoprostatitis, salpingitis, síndrome Cultivo en medio de Thayer- (no en faríngeas)
Fitz-Hugh-Curtis, gonococemia Martin Ciprofloxacino
diseminada (déficit C5-C 8, menstruación,
embarazo, auxotipo AHU)
Uretritis no Chlamydia trachomatis, Similar a las UG, pero con menos Contacto 7-15 días. Excluir gonorrea Tetraciclinas o macrólidos
gonocócica Ureaplasma urealyticum signos y síntomas por Gram y cultivo. C. inclusión-
Epididimitis, proctitis, cervicitis, EIP Giemsa IFD, medios celulares

Tabla 3. Diagnóstico diferencial de las uretritis

En mayores de 35 años el patógeno más frecuente es el E.Coli y el trata-


La Infección nosocomial más frecuente es la infec-
miento sería con quinolonas o cefalosporinas 10-14 días ción de vías urinarias. Y la segunda causa más fre-
cuente de fiebre en pacientes postquirúrgicos.
Absceso renal

Los abscesos medulares o corticales suelen proceder de un foco de El diagnóstico es sim ilar al absceso renal, y su tratamiento pasa por el
pielonefritis contiguo o de diseminación hematógena de S. aureus, drenaje percutáneo o quirúrgico, con la adecuada cobertura antibiótica.
procedente de focos cutáneos en sujetos adictos a drogas por vía
parenteral. El urocultivo en este último caso puede ser negativo. El ITU asociada a catéteres
diagnóstico más fiable se realiza mediante TC. Deben tratarse con an-
tibióticos por vía intravenosa y, dependiendo del tamaño y de la evo- La ITU es la infección hospitalaria más frecuente, y los catéteres urina-
lu ción, se hace obligatorio el drenaje mediante punción percutánea o ríos la principal fuente de sepsis. Se calcula que el 1% de cateterismos
quirúrgicamente. ambulatorios transitorios sufren una ITU posterior y que la mayoría de
pacientes con catéter permanente presentan una bacteriuria significativa
Absceso perirrenal al cuarto día de su co locación. Esta bacteriuria puede hacerse sintomática
en forma de cuadros de cistitis, hematuria o episodios febriles, muchas
Se localiza entre la cápsula renal y la fascia de Gerota. Lo más frecuente es veces autolimitados.
que un absceso cortical se abra a este espacio, pero puede ocurrir tam-
bién por diseminación hematógena. El germen más frecuente es E. coli, y Entre los factores que aumentan el riesgo de ITU asociada a catéter urina-
5. aureus en los casos de diseminación hematógena (Figura 1). río se pueden enumerar: 1) sexo femenino, 2) edad avanzada, 3) mala téc-
nica de sondaje, 4) sistemas de drenaje abiertos y 5) falta de higiene local.

Entre los antibióticos disponibles, parece que las quinolonas son los que
mejor elim inan la película biológica de los catéteres infectados, favore-
ciendo así el tratamiento de la infección; en cualquier caso, este única-
mente se recomienda si existe sintomatología o en el momento de la
retirada del catéter, por el mayor riesgo de ITU sintomática y sepsis.

2.4. Tuberculosis genitourinaria

Genera lmente está ocasionada por Mycobacterium tuberculosis. El apara-


to genitourinario es el sitio más frecuente de afectación extrapulmonar
(tras la adenitis tuberculosa). Un 5% de los pacientes con tuberculosis
activa presentan afectación del tracto genitourinario (Figura 2).

Tras la inhalación del bacilo, se produce una diseminación hematógena


(primoinfección) con siembra de bacilos en ambos riñones en el 90% de
los casos. Sin embargo, la enfermedad clínica generalmente es unilateral.

El periodo de latencia entre la "siembra"y la enfermedad clínica oscila en-


tre 1O y 40 años, afectando principalmente a pacientes por debajo de los
50 años. La lesión inicial microscópica se localiza en los glomérulos en for-
ma de granulomas microscópicos. Al avanzar la enfermedad, se produce
afectación más distal hasta la aparición de una papilitis necrotizante, mo-
mento en el que ya puede existir paso de bacilos a la vía excretora donde,
Figura 1. TC en la que se muestra un absceso perirrenal por procesos inflamatorios, ocasionará estenosis a nivel de los infundí-

02 · Infecciones del tracto urinario.


Cistitis intersticial
ERRNVPHGLFRVRUJ
Urología 1 02
bulos caliciales, pelvis y uréter, con hidronefrosis secundaria. Las lesiones más evolucionado de la enfermedad, el riñón puede encontrarse anula-
renales pueden cavitarse y calcificarse, y llegar a producir una destrucción do, disminuido de tamaño y con calcificaciones parenquimatosas.
total del parénquima (fenómeno que se denomina "riñón mastic").
Tratamiento
El tratamiento médico de la enfermedad activa no difiere sustancialmen-
Obliteración
ureteral ..··~,·-.~
,/' 1 .
te del de la tuberculosis pulmonar en cuanto a fármacos y periodo de
tratamiento. Puede ser necesario el tratamiento quirúrgico, dependiendo
de la complicación asociada, generalmente estenosis de la vía excretora
·,..,;'.:¡~
~·#' e hidronefrosis. Esquema actual de la OMS Rlfampicina + isoniacida + pi-
·t: razinamida + Etambutol por 2 meses, seguido de Rifampicina e isoniacida
por 4 meses, si hay factores de riesgo se extiende tratamiento por 9 a 12
meses. (recurrencia, HIV, inmunosupresión).
Amputación Pionefrosis
calicial

Nemotecnia para tratamiento: PERI (pirazinamida,


Trompa etambutol, rifampicina e isoniacida).
Microvejiga

Próstata
y vesículas
2.5. Cistitis intersticial
seminales

Aunque no es un cuadro infeccioso, se incluye en el presente capítulo


Estenosis
ureteral distal Epididimitis esta entidad inflamatoria vesical de origen desconocido. En este sentido,
se esgrimen dos teorías no demostradas: por un lado, la teoría autoin-
munitaria, y por otro, la de un déficit en el recubrimiento urotelial por
Figura 2. Lesiones de la tuberculosis genitourinaria glucosaminoglucanos.

Clínica Clínica
Los hallazgos clínicos son escasos. En el 70% de los pacientes, los sínto- Suele presentarse en mujeres entre 30 y 70 años, como un cuadro cistí-
mas son leves. Lo más frecuente es la aparición de microhematuria, dolor tico crónico en el que destacan disuria, polaquiuria con nicturia y dolor
vago en flanco o cólico renal. La afectación vesical, sin embargo, sí produ- suprapúbico, acompañados en ocasiones de hematuria (20-30%).
ce sintomatología florida con un síndrome cistítico rebelde, donde la po-
laquiuria (secundaria a la disminución de la capacidad vesical) es lo más
llamativo. En varones, es frecuente la aparición de una orquiepididimitis Existen muchas más causas de síndrome cistítico: cis-
titis aguda, tuberculosis, carcinoma in situ, etcétera.
crónica que no responde a la terapia habitual.

En el 90% de los pacientes, el análisis urinario es anormal. Típicamente apa-


rece piuria ácida con urocultivo negativo. La prueba de laboratorio más
importante es el cultivo de M. tuberculosis en medio selectivo (Lówenstein), Diagnóstico
ya que los medios de tinción rápida (Ziehl, aura mina), aunque válidos, pue-
den dar falsos positivos por contaminación con M. smegmatis. El diagnóstico es básicamente por exclusión de otra patología que pueda
ocasionar un cuadro similar (infección bacteriana, tuberculosis, litiasis o
Diagnóstico tumor vesical) apoyado en los hallazgos cistoscópicos sugestivos: 1) pe-
tequias submucosas, principalmente trigonales, que aparecen al disten-
El cultivo en medio de Lówenstein es positivo en el 90% de los pacientes der la vejiga (glomerulaciones), 2) úlceras de Hunner. La biopsia vesical,
con enfermedad activa, aunque deben obtenerse, al menos, tres mues- además de descartar la presencia de carcinoma in situ u otra patología,
tras de días diferentes para mejorar la sensibilidad, ya que el paso de ba- revela en algunos casos, un infiltrado intersticial de mastocitos.
cilos a orina no es constante. Además el resultado tarda de 1 a 3 meses.
Actualmente, lo más rentable es realizar una PCR de orina en busca del Tratamiento
ARN del bacilo. Hasta 80% tienen pruebas cutáneas positivas.
Aunque esta enfermedad raramente supone una amenaza para la vida
Radiológicamente, el 90% de los pacientes presentan urogramas altera- de la paciente, su morbilidad es elevada. Desgraciadamente, las diversas
dos. El hallazgo más sugestivo es la presencia de cavidades que comu- alternativas de tratamiento únicamente pueden encaminarse a una me-
nican con el sistema colector. Inicialmente estas cavidades son mínimas joría sintomática, en la mayoría de los casos con resultados discretos; 1)
y dan un aspecto "mordisqueado" a los cálices. Según la enfermedad distensión hidráulica vesical, 2) amitriptilina oral, 3) instilación con dime-
avanza, pueden encontrarse estenosis infundibulares, ureteropiélicas, en tilsulfóxido [DMSO], 4) corticoides tópicos o sistémicos, 5) denervación
unión ureterovesical o vejigas pequeñas de aspecto rígido. En el punto vesical, 6) cistoplastias de aumento, y en último término, 7) cistectomía.

ERRNVPHGLFRVRUJ
M an ual CTO de Medicina y Cirugía, 3.ª edición

" El diagnóstico definitivo de ITU es microbiológico: más de 105


Ideas clave UFC/ml. No obstante, este criterio varía con el sistema de reco-
gida.
" La causa más frecuente de infección del tracto urinario (ITU) es
Escherichia coli, tanto a nivel comunitario como nosocomial. " Si se recoge la muestra urinaria mediante punción suprapúbica,
cualquier número de bacterias es significativo.
" El origen más frecuente de uretritis es Chlamydia trachomatis.
" La bacteriuria asintomática se trata en gestantes, menores de 5
" La causa habitual de orquiepididimitis depende de la edad: años, inmunodeprimidos, previamente a la cirugía urológica, o
Chlamydia y gonococo si es menor de 35 años; enterobacterias, si la especie implicada es Proteus.
si es mayor de esa edad.
" Los sistemas de drenaje cerrados son preferibles a los abiertos,
" La causa más frecuente de absceso renal en el UDVP es Sta- pues la tasa de infección es menor.
phylococcus aureus.

Un paciente de 83 años sondado de forma permanente acude a


Casos clínicos la consulta tras detectársele dos cultivos positivos tomados con
una semana de diferencia. Asegura encontrarse asintomático. La
Un prostático, sin otros problemas de salud, portador de sonda actitud más adecuada será:
uretral permanente, presenta bacteriuria (> 105 unidades for-
madoras de colonias) en dos urocultivos. ¿Cuál es la actitud tera- 1) Iniciar tratamiento antibiótico según antibiograma de los culti-
péutica más conveniente? vos obtenidos.
2) Tranquilizar al paciente y seguir con su pauta habitual de re-
1) Tratamiento antibiótico de amplio espectro. cambio de sonda.
2) Tratamiento antibiótico según antibiograma. 3) Realizar cambio de sonda de forma inmediata con tratamiento
3) Cambio de sonda urinaria exclusivamente. antibiótico.
4) Antisépticos en vejiga urinaria. 4) Realizar cambio de sonda de forma inmediata con profilaxis an-
tibiótica de 4 días.

Ante un paciente de 24 años, que presenta fiebre alta con dolor,


inflamación y enrojecimiento testicular izquierdo, ¿cuál de las si- Hombre de 54 años, con cuadro clínico de 6 meses de evolución
guientes afirmaciones es INCORRECTA? caracterizado por disnea de medianos esfuerzos; en últimas 6 se-
manas fiebre, astenia, adinamia, y pérdida de peso y 4 días antes
1) El diagnóstico más probable es el de epididimitis. disuria y polaquiuria. La radiografía de tórax muestra derrame
2) Los patógenos más frecuentes son Chlamydia trachomatis y pleural derecho e infiltrados algodonosos generalizados. Prue-
Neisseria gonorrhoeae. ba de VIH negativa. Se realiza urocultivo el cual resulta negativo,
3) El tratamiento de elección es vancomicina + gentamicina. examen general de orina reporta microhematuria. ¿Cuál es el
4) El tratamiento de elección puede ser otloxacino. diagnostico más probable?

1) Pielonefritis.
Ante un paciente que presenta febrícula persistente, crisis 2) Bacteriuria asintomática.
renoureterales breves, piuria estéril, orina con pH ácido, mi- 3) Tuberculosis extrapulmonar.
crohematuria persistente, con citología urinaria negativa y 4) Neumonía adquirida en la comunidad.
epidídimos indurados ¿en qué enfermedad se debe pensar
primero?
Con base a su diagnóstico ¿Cuál es la duración del tratamiento?
1) Sarcoidosis.
2) Carcinoma vesical. 1) 9 meses.
3) Carcinoma renal. 2) 6 meses.
4) TBC urogenital. 3) 12 meses.
4) 24 meses.

Una paciente de 27 años acude al servicio de Urgencias por do-


lor en fosa renal derecha, fiebre de 39 °C, escalofríos y síndrome Femenino de 29 años de edad, cursando con embarazo de 12
miccional acompañante. Es alérgica a penicilinas. Señale la res- semanas de gestación, examen de rutina muestra infección de
puesta correcta: vías urinarias, paciente niega sintomatología urinaria. ¿Cuál es la
conducta a seguir?
1) No será necesario descartar patología urinaria obstructiva en
este caso, ya que presenta un claro síndrome miccional. 1) Hallazgo incidental, no necesita tratamiento.
2) Para poder hacer el diagnóstico de pielonefritis se deberá cono- 2) Tratamiento con amoxicilina durante 3 días.
cer primero los datos referidos a la función renal. 3) Tratamiento con amoxicilina durante 7 días.
3) Se deberá iniciar tratamiento empírico con un ~-lactámico. 4) Se recomienda a la paciente que consuma jugo de arándano y
4) Si en las primeras horas evoluciona favorablemente, podrá con- se prescribe acido ascórbico.
tinuar el tratamiento de forma ambulante.

02 · Infecciones del tracto urinario.


Cistitis intersticial
ERRNVPHGLFRVRUJ
Urología 1 02
brought to the emergency department complaining of hematu-
Case Study ria, high fever, chills and left flank pain. In the previous days, she
presented dysuria, polyakyuria which made her recall a previous
A 52-year-old man presents to the emergency department with cystitis episode, so she took fosfomycin. In this patient, which of
complaints of fever, chills, perineal pain and pallor. In the mor- the following options is correct?
ning, he underwent a transrectal prostate biopsy. On physical
examination, is found persistent low blood pressure and high 1) An abdominal x-ray must be performed to rule out an obstruc-
fever. His laboratory tests reveal elevated WBC count. Which of tive cause.
the following sentences is incorrect? 2) She may receive a 21-days treatment with levofloxacin.
3) A fetal wellness test must be performed.
1) Hemodynamic stabilization is mandatory. 4) In pregnant women, cystitis can lead to acute pyelonephritis in
2) This clinical condition is very rare after a transrectal biopsy, sin- 10% of cases.
ce it isn't a frequent complication ofthis procedure.
3) Long-term antibiotic treatment is indicated.
4) lt's unlikely that he needs to undergo a surgical procedure to Which of the following is the best diagnostic test for genitouri-
treat his disease. nary tuberculosis?

1) Agar-agar culture.
A 23-year-old woman sees the physician because a urine culture 2) Ziehl-Nielsen staining.
performed a week ago, shows > 10,000 CFU/ml of non-resistant 3) Urine PCR test.
E. coli. Currently, she hasn't symptoms, and claims that she had 4) Lowenstein culture.
cystitis once when she was a teenager. Blood tests reveals: crea-
tinine 0.7mg/dl, WBC count 9,000/mm3 and hemoglobin 13.29/
dl. Pregnancy test is positive. Which of the following is the co- Which of the following is not true regarding recurrent urinary
rrect management of this patient? tract infections?

1) Treatment is not necessary, since it's an asymptomatic bacteriuria. 1) lt's diagnosed when a patient presents four or more episodes
2) She will probably develop symptoms in a few days. peryear.
3) She needs treatment, given the high likelihood of developing acu- 2) lt usually appears in young male patients who are sexually ac-
te pyelonephritis. tive.
4) Ciprofloxacin S00mg/q 12 hours may be an effective treatment 3) Sometimes it may be associated with Sthaphylococcus sapro-
option. phyticus.
4) The first step in management consists in taking hygiene mea-
sures.
A 23-year-old woman, who is in her 13th week of pregnancy, is

ERRNVPHGLFRVRUJ
_ _ _ _ _Urolo_gía_

Urolitiasis

Tema fundamental en esta asignatura. Se debe conocer muy bien la actitud ante la litiasis
en general, ante los distintos tipos de cálculos y, especialmente, todo lo relacionado con el
tratamiento. Es un tema rentable y agradecido, así que hay que emplear el tiempo necesario.
La tabla-resumen de urolitiasis puede ser de gran ayuda.

3.1. Epidemiología La enfermedad litiásica recidiva en el 40% de los casos, con una media
de un nuevo cálcu lo cada dos o tres años. Por recidiva se entiende la
aparición de una nueva litiasis de la misma composición y en la misma
La litiasis del aparato urinario es una patología frecuente en la población localización, en un intervalo menor de cuatro años entre un cálculo y
mundial, y en México representa un problema endémico, constituyendo otro.
una causa de consulta y de atención urgente con potencial daño estruc-
tural y repercusión funcional en todo el aparato urinario.
3.2. Manifestaciones clínicas
Son diversas las moléculas que forman parte de los cálculos. Su incidencia va-
ría según el país, e incluso según las áreas geográficas dentro del mismo país.
y su manejo agudo
Se pueden distinguir seis grupos de componentes: El dolor agudo del cólico renal es la manifestación más típica de la litiasis
Oxalato cálcico. renal. El dolor se produce por la sobredistensión de la vía urinaria tras la
Fosfato cálcico. obstrucción de esta por el cálculo. Es lógico, por tanto, que el cálculo
Fosfato no cálcico. deba desplazarse desde su origen calicial para producir sintomatología
Compuestos purínicos (ácido úrico, urato amónico, urato sódico, aguda. Ocasionalmente se observan cuadros de dolor vago renal en rela-
xantina, 2,8 dihidroxiadenina). ción con litiasis caliciales no desplazadas.
Aminoácidos (cistina).
Otros (carbonato cálcico, sulfamidas, etcétera). El cólico renal o crisis renoureteral suele aparecer de forma progresiva
sobre la fosa lumbar afectada, irradiándose por el flanco hacia la ingle y
Los cálculos de oxalato cálcico son los más frecuentes, con cifras en torno los genitales (Figura 1). El paciente generalmente se encuentra afectado,
al 65%, seguidos por los infecciosos y ácido úrico (alrededor del 15% cada con dolor que no cede con reposo, por lo que cambia de postura con-
uno). Fosfato cálcico un 5%, y los de cistina con una incidencia baja (1-3%). tinuamente. Puede acompañarse de un cortejo vegetativo con náuseas,
vómitos y diaforesis.
La tercera década es la edad media de aparición, por primera vez, de la
litiasis salvo en los de cistina, que suelen ser de aparición más prematura. El dolor irradiado hacia la ingle generalmente indica que el cálculo ha
Se debe de tener en cuenta los factores de riesgo para la formación de alcanzado el uréter. Cuando se encuentra en vecindad de la vejiga, o bien
cálculos: Antecedentes familiares, infecciones, hiperparatiroidismo, enfer- dentro de esta, puede aparecer un cuadro irritativo, similar al síndrome
medades gastroeintesitnales, alteraciones anatómicas, entre otras. miccional con polaquiuria, disuria y tenesmo vesical.

Los cálculos infecciosos de estruvita, y en menor medida, los de ácido


úrico y cistina pueden crecer modelando las cavidades renales (litiasis
Las infecciones urina ri as son más frecuentes en muje-
res que en varones. Por eso los cálculos de estruvita coraliforme o "en asta de venado" [Figura 2]), manifestándose no como
también lo son. cólico, sino como infecciones urinarias de repetición, dolor lumbar sordo,
hematuria o incluso insuficiencia renal terminal.

ERRNVPHGLFRVRUJ
Urología 1 03
En teoría, el 70% de los cálculos son visibles en una radiografía simple
de abdomen, aunque este porcentaje es considerablemente menor en
las radiografías urgentes sin preparación intestinal. Radiológicamente,
la mayoría de los cálculos son radioopacos, exceptuando los de ácido
úrico y algunas otras composiciones infrecuentes (sulfamidas, xantina,
indinavir).

El estudio de imagen se completará mediante otras técnicas diagnósti-


RX simple de abdomen cas. La ecografía permitirá visualizar incluso las litiasis radiotransparentes,
con cálculo a nivel de L3
con el inconveniente de no ser vistas aquellas ubicadas en el trayecto
ureteral (salvo las zonas cercanas a la vej iga o al riñón). También se podrá
eval uar el grado de hidronefrosis.

La urografía ofrece información morfológica y funcional de ambos ri-


ñones (Figura 3). Debe tenerse en cuenta que, durante el cólico renal,
puede observarse una anulación funcional, sin que sign ifique nece-
sariamente deterioro de dicha unidad renal. Mediante esta técnica se
Dolor irradiado Síndrome miccional
Hematuria puede diagnosticar todo tipo de cálculos, ya sean radiotransparentes
o radioopacos. El principal inconveniente de este procedimiento es la
introducción de contraste yodado, que está contraindicado en los pa-
cientes con alergia, creatinina mayor de 2, mieloma múltiple o deshi-
dratación importante.
Figura 1. Diagnóstico de urolitiasis

Figura 3. (a) Litiasis ureteral. (b) UIV de uropatía obstructiva izquierda


Figura 2. Litiasis coraliforme o "en asta de venado"
Según las guías clínicas, la urografía intravenosa (UIV) actualmente ha
Diagnóstico sido desplazada por la TC helicoidal sin contraste, que se ha convE:,tido
en el nuevo estudio de referencia para las litiasis. Aunque su alto coste
El análisis básico de orina muestra generalmente hematuria y leu- hace que todavía no esté extendido su uso, permite evaluar todo tipo de
cocituria. Una piuria importante apoyaría la posibilidad de infección cálculos.
sobreañadida, aunque ninguno de estos datos es realmente deter-
mi nante. En niños y embarazadas la ecografía urinaria es la técnica de imagen de
elección.
Exámenes básicos a realizar en urgencias a pacientes con cólico renal:
Urianalisis. Tratamiento
Análisis de sedimento urinario y/o urocultivo.
Electrolitos séricos y creatinina sérica. El manejo agudo del cólico renal se basa en el control del dolor. Para
Biometria hemática. esto, es preciso conseguir una disminución de la presión dentro de la
vía urinaria, lo que puede hacerse, sobre todo, con antiinflamatorios, que
El análisis de la composición de los cálculos se debe realizar en todos disminuyen el dolor y la diuresis al inhibir la síntesis de prostaglandinas.
los cuadros iniciales, las técnicas para hacerlo son con espectroscopia o Asimismo, se pueden usar espasmolíticos, que disminuyen la presión in-
difracción de rayos x. traureteral al relajar la pared del uréter.

ERRNVPHGLFRVRUJ
Manual CTO de Medicina y Cirugía, 3.ª edición

Recomendaciones para cuadro agudo: primera elección diclofenaco, in- 3.3. Evaluación y tratamiento
dometacina o ibuprofeno. Se pueden utilizar alfa bloqueadores para re-
ducir recurrencia de cuadros
de la litiasis renal

Existen una serie de situaciones en las que el cólico renal se convierte en Este apa rtado se puede dividir en dos partes. Por un lado, el estudio de
una urgencia que precisa de hospitalización, y eventualmente, de mani- la litiasis con la finalidad de instaurar un tratamiento preventivo de su
pulación invasiva: formación, y por otro, el estudio y tratamiento de la litiasis ya formada.
Obstrucción grave, principalmente si se acompaña de litiasis mayor
de 1O mm. u obstrucción bilateral. Estudio y tratamiento preventivo
Dolor incontrolable.
Riñón único. La evaluación del paciente con litiasis se basa en un estudio metabólico
Sepsis de origen urológico. para determinar qué factores son modificables, en un intento de evitar la
Falla renal aguda o anuria. recidiva (Tabla 1).
Mujer gestante.

-
Asimismo, en pacientes diabéticos, por el mayor riesgo de complica-
Litiasis cálcica Litiasis
ciones, es aconsejable, si no el ingreso, al menos una observación es- (oxalato o fosfato) úrica
tricta.
Hipercalciuria Gota primaria Cistinuria Infecciones
Una situación similar ocurre durante el embarazo, donde una dilatación idiopática Hemopatías por gérmenes
leve de la vía urinaria puede considerarse"fisiológica': pero obstrucciones Hipercalciuria productores
Enfermedades
secundaria de ureasa
más importantes o la aparición de fiebre hacen aconsejable la colocación digestivas
a hipercalcemia
de un catéter ureteral (Figura 4). lngesta
Hiperuricosuria excesiva
Hiperoxaluria de purinas
Hipocitraturia Fármacos
Acidosis renal tubular Litiasis úrica
distal idiopática
Litiasis cálcica
idiopática

Tabla 1. Tipos de litiasis. Situaciones que favorecen su aparición

Este estudio debe reservarse para aquellos pacientes con alta probabili-
dad de recidiva, aunque cada vez más autores indican que debe realizar-
se a todos los pacientes (Tabla 2).

La furosemida, al revés que las tiacidas, aumenta el


calcio urinario.

Edad temprana de aparición


Litiasis bilateral
Litiasis en riñón único o malformado
Composición poco frecuente
Litiasis recidivante
Nefrocalcinosis
Litiasis coraliforme
Tabla 2. Pacientes con indicación de estudio metabólico

Desde el punto de vista práctico, las litiasis se pueden dividir en: las de
composición cálcica y las de otras composiciones, ya que el primer grupo
supone la mayoría de los casos (70-80%) tratados habitualmente.

Litiasis cálcica

Figura 4. Doble J derecho. Litiasis ureteral derecha. Litiasis coraliforme En la mayoría de las ocasiones se desconoce el origen de la litiasis cálcica,
izquierda aunque se puede hacer una aproximación a los factores de riesgo que

0 3 · Urolitiasis
ERRNVPHGLFRVRUJ
Urología 1 03
influyen en su aparición. Sólo en un pequeño porcentaj e de casos existe Acidosis tubular renal dist al (véase Sección de Nefrología). En-
una enfermedad de base que puede ser tratada, y de esta forma desapa- fermedad autosómica recesiva . Consiste en la imposibilidad del
rece la formación de cálcu los cá lcicos. túbulo distal para excretar hidrogeniones a la orina (orinas persis-
Hipercalciuria idiopática. Es la causa más frecuente de litiasis cál- tentemente alca li nas) con aumento de la eliminación de calcio a
cica. Se define como una excreción urinaria de ca lcio mayor de 300 la orina.
mg/24 h en el varón y 250 mg/24 h en la mujer. De cara a su manejo, Existen formas incompletas que se observan en pacientes formado-
las tiazidas disminuyen el calcio urinario, reduciendo la formación res de cálculos de oxalato cálcico y con hipercalciuria idiopática. En
de litiasis. La administración de citrato potásico ayuda a evitar la hi- estos probablemente la acidosis tubular no juegue un papel impor-
popotasemia y aumenta el citrato urinario, que es inhibidor de la tante y responden a tiazidas.
litogénesis (Tabla 3). Otras ci rcunstancias que favorecen la litiasis cálcica son: sarcoi-
dosis, síndrome de Cushing, diuresis escasa, déficit de inhibidores o
anoma lías en el pH urinario (alcalosis).
Absortivas Resortivas Renales Litiasis cálcica idiopát ica. Aproximadamente en el 20% de los pa-
cientes con litiasis cá lcica no se demuestra ninguna anomalía en el
Aporte excesivo Hiperparatiroidismo Acidosis tubular
distal estudio metabólico.
Sd. de Burnett (leche Inmovilización
y alcalinos) Tumorales ldiopática
Hipervitaminosis O Enf. Paget
Litiasis úrica
ldiopática Sd. de Cushing
Sarcoidosis El ácido úrico no disociado es poco solu ble en orina. Con un pH urinario
de 5, la solu bilidad del ácido úrico es únicamente de 100 mg/I, mientras
Tabla 3. Causas más frecuentes de hipercalciuria que con un pH de 7 es de 1.580 mg/1. Esto demuestra la gran importan-
cia del pH urinario en la formación de cálculos de ácido úrico. Aparte
Hiperuricosuria. Excreción en orina de más de 800 mg/24 h en el de estos, también existe una pequeña proporción de cálculos de urato
varón o 750 mg/24 h en la mujer. Además de favorecer la litiasis úri- monosódico y urato amónico.
ca, la hiperuricosuria constituye un factor de riesgo para la formación
de cálculos de calcio, probablemente por nucleación heterogénea El objetivo del tratamiento es reducir el ácido úrico excretado y aumen-
sobre núcleos de ácido úrico o urato sódico. Generalmente se debe tar el pH urinario, ya que los cálculos más frecuentes en pacientes hipe-
a un exceso de purinas en la dieta. ruricémicos son los de ácido úrico. Por otra parte, este tipo de cálculos
Hiperoxaluria. Se considera como tal la excreción en orina de son los que mejor responden al tratamiento médico mediante quimió-
más de 40 mg/24 h. Existe una hiperoxaluria primaria, que es con- lisis por alcalinización urinaria. Pueden adm inistrarse diversos álcalis; el
secuencia de un defecto enzimático autosóm ico recesivo; no tiene citrato potásico impediría el teórico riesgo de formación de cálculos
tratamiento y generalmente conduce a insuficiencia renal por litiasis cálcicos por su efecto inhibidor, pero también pueden tratarse con bi-
recidivante. El único tratamiento que existe actualmente es el tras- carbonato sódico o citrato sódico. Una alternativa es la acetazolamida
plante hepático, que suele ir unido al renal, aunque algunos casos en dosis de 250 mg/día. Cuando, además, la uricemia es alta, puede
responden a piridoxina. tratarse con alopurinol.

Litiasis cistínica
La causa más frecuente de hipercalcemia en un pa-
ciente ambulatorio es el hiperparatiroidismo prima- La cistinuria es un trastorno autosómico recesivo en el que existe un de-
rio. En cambio, la hipercalcemia más frecuente en
fecto de absorción, a nivel intestinal y tubular proximal, de los aminoá-
uno ingresado es la de origen neoplásico.
cidos dibásicos: cistina, ornitina, lisina y arginina (COLA), aunque parece
que puede existir un trastorno en el que únicamente se ve afectada la
No obstante, la mayoría de los casos de hiperoxaluria son secundarios cistina, lo que indicaría que, además de un mecanismo de transporte co-
a malabsorción de ácidos grasos por enfermedades crónicas pancrea- mún, existe uno independiente para la cistina.
tobiliares, derivación intestinal para el tratamiento de la obesidad mór-
bida, resección ileal, enfermedad inflamatoria intestinal, hipercalciuria Los niveles de cistina en orina de 24 horas son superiores a 100 mg, de
coincidente o por falta de calcio en la dieta, lo que permite que exista hecho, los homocigotos pueden excretar más de 600 mg/día. El diagnós-
mayor cantidad de oxalato intestinal para su absorción. tico se realiza identificando los característicos cristales hexagonales en
La intoxicación por etileng licol y metoxiflurano puede producir hi- orina, o por una prueba positiva de nitroprusiato sódico (la orina se tiñe
peroxaluria, así como la ingesta de vitamina C en altas dosis. En todos de azul en pacientes afectados por esta enfermedad: test de Brand).
estos casos secundarios, el tratamiento con colestiramina, una dieta
pobre en grasas y la correcció n de la malabsorción, en la medida de El tratamiento consiste en aumentar la diuresis diaria (más de 3 1/día),
lo posible, suelen ser medidas eficaces. alcalinizar la orina por encima de 7,5 y, en caso de que esto sea insufi-
Hipocitraturia. Excreción de citrato inferior a 300 mg/24 h. General- ciente, puede iniciarse tratamiento con D-penicilamina (250 mg/6 h) o
mente se asocia a otras anoma lías urinarias. Aunque de causa des- u-mercaptopropionilglicina (250 mg/6 h).
conocida, puede contribuir una dieta rica en proteínas, hipocaliemia,
enfermedad intestinal o infección urinaria. Litiasis infecciosa
Hiperparatiroidismo primario. Supone la causa más frecuente de
hipercalciuria conocida (véase Sección de Endocrinología, metabolis- Los cálculos infecciosos de estruvita o de fosfato amónico magnésico
mo y nutrición). (MgNH/O4-6Hp) se desarrollan en un ambiente alcalino, producido por

ERRNVPHGLFRVRUJ
Manual CTO de Medicina y Cirugía, 3.ª edición

infección persistente de gérmenes que hidrolizan la urea, aumentando A continuación se analizan brevemente las diversas formas de trata-
la cantidad de amonio urinario. Los principales gérmenes que poseen miento:
ureasa, además de diversas especies de Proteus, son Pseudomonas, K/e- Cirugía. Ha sido el tratamiento estándar hasta la aparición de la
bsiella, Serratia y Enterobacter. La presencia de cuerpos extraños (sondas litotricia extracorpórea. Aún hoy día, es preciso recurrir a la cirugía
vesicales, suturas) favorece su formación. cuando fracasan las ondas de choque o en determinados casos para
reducir la masa litiásica (cálculos coral iformes).
Para su tratamiento se han empleado diversos métodos, generalmente Endourolog ía. La manipulación endoscópica de la vía urinaria es
ineficaces. La antibioterapia únicamente mantiene estéril la orina durante cada día más accesible gracias a las mejoras técnicas. Puede realizar-
los cursos de tratamiento. se extracción directa del cálculo mediante diversos tipos de pinzas o ·
cestillas, o bien fragmentar previamente el cálculo mediante diversas
Parece más prometedor el uso de inhibidores de la ureasa con ácidos fuentes de energía, como la electrohidráulica, ultrasónica o láser. Se
hidroxámicos. Estos son moléculas análogas a la urea que forman un puede acceder hasta el cálculo mediante ureterorrenoscopia (URS) o
complejo enzima-inhibidor irreversible. Se utilizan básicamente dos nefrolitotomía percutánea (NLPC).
sustancias de esta naturaleza: el ácido propiónico y el acetohidroxá- Litotricia extracorpórea por ondas de choque (LEOC). Las on-
mico. das de choque se transmiten a través de los tejidos corporales con
la misma impedancia acústica que el agua hasta alcanzar la litiasis,
Su empleo suele venir acompañado de cefaleas, temblores, trombosis sobre la que produce fenómenos de compresión y descompresión
venosas u otros síntomas neurológicos, por lo que tampoco son de gran que conducirán a su fragmentación.
aceptación. La gran mayoría de los cálculos renales son susceptibles de trata-
miento mediante LEOC. Las únicas limitaciones serían aquellos cál-
Todo lo relativo al estudio de la nefrolitiasis expuesto anteriormente se culos no localizables por su pequeño tamaño (< 2-5 mm), dureza
puede repasar en la Tabla 4. del calculo mayor de 1000 UH u obesidad, también particularidades
anatómicas de la vía excretora y paciente, función renal y tipo de
Tratamiento de la litiasis ya formada (Figura 5) litotriptor disponible.

Los cálculos ya formados no expulsables (> 6 mm) precisan de trata- La presencia de hipertensión arterial no controlada facilita el riesgo de
miento "agresivo", es decir, necesitan ser extraídos quirúrgicamente o hemorragia durante la sesión de litotricia, luego deberá ser estabilizada
fragmentados de forma que puedan ser expulsados espontáneamen- previamente a la misma y constituye, en cierto modo, por ello, contrain-
te. dicación relativa de LEOC (Tabla 5).

Estruvita (fosfato
Sales cálcicas Ácido úrico Cistina
amónico magnésico)
Frecuencia · Oxalato cálcico: 55-60% 10-15% 5-10% 1-3%
· Fosfato cálcico: 10-15%
Sexo Varón Mujer Varón Varón= Mujer
Etiología Hipercalciuria id iopática Infección por gérmenes ureasa Gota (50%) Cistinuria
ldiopática (+) ldiopática (< 50%)
Hiperuricosuria (20%) Hiperuricemias
secundarias
pH Alcalino Alcalino Ácido Ácido
Radiología Radioopacos Radioopacos Rad iotra nspa rentes Radiolúcidos
Morfología Formas prismáticas Aglomerados de cristales Cristales hexagonales
de los cristales polimórficas desorganizados, en prismas o láminas
Cristales "en ataúd" a veces formando masas
continuas
Cristales de fostato Ca

Cristales de Oxea Cristales de estruvita Cristales de ácido úrico Cristales de cistina


Tratamiento Hipercalciuria idiopática: Ácido propiónico y ácido Alcalinizar la orina Forzar diuresis (ingesta
tiazidas acetohidroxámico Alopurinol hídrica)
Hiperoxaluria 1.ia: piridoxina Antibioterapia (si hay hiperuricemia) Alcalinizar orina
Hiperoxaluria 2.ia: En ocasiones cirugía Dieta de bajo contenido D-penicilamina
colestiramina proteico (si no hay respuesta)
Tabla 4. Tabla-resumen de las nefrolitiasis

03 · Urolitiasis
ERRNVPHGLFRVRUJ
Urología 1 03
M,in e¡o de la uroliti.1so ,

Crisis Estable
cuadro agudo cuadro cronico

¿Edad?
Indican
¿Tipo de cálculos?
o noLEOC
¿Periodicidad de la clínica?
o cirugía
¿Tipo de síntomas?
¿Viabilidad renal?

ratamlento slntom6tlco: · Obstrucción grave


· Espasmolltlcos Tratar la condición
· Infección, fiebre Litotricia Cirug ía
y antllnflamatorlos preexistente
· Dolor incoercible
· Reposo e hidratación · Riñón único t
· Cálcica: acidificar (no útil si oxalato),
t
Ecografía renal
citratos, tiazidas colestiramina
Extracorpórea
(LEOC)
· Ureterotomía
· Pielolitotomía
y dieta baja en grasas y rica · Percutánea · Nefrectomia
t en calcio, si hiperoxaluria
· Úrica: alcalin izar, alopurinol
ultrasonográfica
con microlumbotomía
Dilatación
· Estruvita: acetohidroxámico · Endoscópica
Ingreso y tratamiento agresivo: · Cistina: D-penicilamina, vit. 86 vía ureteral
· Drenaje y alcalinizar
(catéter o nefrostomla)
Contraindicada en:
· Tratamiento parenteral:
· Embarazo
- Antibióticos
· Infección
- Remontar hemodlnámica
· Obstrucción distal
- Equilibrio electrolltlco
· Aneurismas
- Narcóticos · Coagulopatlas
· Obesidad
· Vigilancia estrecha
· Arritmia cardiaca

Figura S. Manejo de la litiasis renal ya formada

Relativas (precisan de control Esta posibilidad es mayor ante litiasis de gran tamaño, por lo que en al-
Absolutas
previo al tratamiento)
gunos de estos casos se puede colocar un catéter de derivación urina ria
Embarazo Alteraciones de la coagu lación (nefrostom ía o doble J) antes de la LEOC para disminuir este riesgo, ge-
Obstrucción distal Aneurisma aórtico nera lmente en litiasis superiores a 2 cm.
Infección activa Alteraciones del ritmo ca rdíaco,
ma rca pasos o desfi bril adores Derivadas del efecto directo de las ondas de choq ue, pueden aparecer
Obesidad contusiones renales ma nifestadas como hemat uria, hematomas rena les,
Hipertensión arterial descontrolada equ imosis o eritema cutáneo, y en grado máximo, rotura renal. La hema-
Tabla S. Contraindicaciones de la LEOC tu ria se considera la comp licación más frecuente de la litotricia.

Complicaciones Más controvertida es la teó ri ca relación de la LEOC con la aparición de


hi pertensión arterial, ya que no está demostrada en las últimas revisiones
La expulsión de fragmentos litiásicos puede ocasionar un cólico rena l y, publicadas, aunque sí la relación entre hematoma renal post-LEOC e hi-
con menor frecuencia, obstrucción ureteral (steinstrasse o "ca lle litiásica"). pertensión arterial.

" Los cálculos asociados a las resecciones ileales o a la enferme-


1
Ideas clave dad inflamatoria intestinal son de oxalato cálcico.

" Los cálculos más frecuentes son los de oxa lato cálcico. " Precipitan en medio ácido: ácido úrico y cistina. Precipitan en
medio alcalino las que contienen fosfatos (fosfato amónico
" Globalmente, la litiasis es más común en el varón, salvo las de magnésico o estruvita, y el fosfato cálcico).
estruvita, más comunes en mujeres.
" En el tratamiento de la litiasis por ácido úrico es beneficioso al-
" La radiografía de abdomen no permite ver algunos cálculos, calinizar la orina.
como los de urato. Sin embargo, la ecografía puede verlos, inde-
pendientemente de su composición. " Los cálculos de oxalato NO se ven alterados por el pH (al Oxal,
el pH de la igual).
" Litiasis radiotransparentes: Sulfamidas, lndinavir, Urato, Xanti-
nas (SIUX) . Las de cistina son radiolúcidas; y el resto, radioopa- " Los cálculos de estruvita se relacionan con microorganismos
cas. p roductores de ureasa, como Proteus.

" Las tiazidas son útiles para la hipercalciuria idiopática. " Las contraindicaciones absolutas para la LEOC son: embarazo, in-
fección activa y obstrucción de las vías urinarias distal al cálculo.

ERRNVPHGLFRVRUJ
Manual CTO de Medicina y Cirugía, 3.ª edición

Masculino de 26 años de edad ingresa al servicio de urgencias


Casos clínicos por presentar dolor tipo cólico localizado en fosa renal derecha,
acompañado de fiebre, nauseas y vomito en 4 ocasiones, ade-
A un hombre de 29 años, con antecedentes de dolor tipo cólico más de presentar hematuria franca. ¿Cuál es el tratamiento indi-
en fosa renal izquierda que cedió con tratamiento analgésico, se cado en este caso?
le practica una urografía intravenosa, apreciándose defecto de
repleción radiotransparente de 6x7 mm en tercio distal de uré- 1) Manejo ambulatorio con ketorolaco, butilhioscina y metoclo-
ter izquierdo. El pH de la orina fue de 5,5; asimismo, se observan pramida.
cristales de urato, 9-12 hematíes por campo y escasa leucocitu- 2) Hospitalización, ketorolaco, antiespasmódico y diurético.
ria. ¿Cuál sería el tratamiento más apropiado? 3) Hospitalización diclofenaco, antiespasmódico.
4) Manejo ambulatorio con paracetamol, antibioticoterapia empí-
1) Alopurinol vía oral. rica y cita a la consulta externa.
2) Ureteroscopia con extracción del cálculo.
3) Nefrolitotomía endoscópica percutánea.
4) Alcalinización de la orina por vía oral. ¿Cuál es el estudio de imagen de elección?

1) TAC de abdomen.
Mujer de 50 años, diabética insulinodependiente, con infeccio- 2) Placa de abdomen.
nes urinarias y cólicos nefríticos de repetición. Acude a Urgen- 3) USG renal bilateral.
cias por dolor en fosa renal izquierda de cinco días de evolución, 4) Urografía excretora.
asociado en las últimas 24 horas a fiebre, escalofríos y malestar
general. Analítica de sangre: plaquetopenia, leucocitosis y dis-
minución de la actividad de la protrombina. Analítica de orina ¿Qué estudios de laboratorio solicitaría en este caso?
normal. Radiografía de abdomen con claras imágenes de litiasis.
Eco renal: dilatación moderada de sistema excretor izquierdo. 1) Biometría hemática, química sanguínea, tiempos de coagula-
¿Cuál es la conducta más adecuada? ción, proteína e reactiva, urocultivo.
2) Biometría hemática, examen general de orina, electrolitos seri-
1) Solicitar hemocultivos y urocultivo para establecer la necesidad cos, química sanguínea.
de antibioterapia. 3) Biometría hemática, examen general de orina, química sanguí-
2) Realizar urografía intravenosa para intentar filiar la causa. nea.
3) Hidratar a la paciente bajo observación rigurosa, y repetir eco- 4) Examen general de orina, proteína e reactiva, tiempos de coa-
grafía a las 48 horas. gulación, biometría hemática.
4) Colocar catéter doble J o practicar nefrostomía percutánea de
forma inmediata con cobertura antibiótica.

1) The first diagnostic test to perform in this patient is a contrast


Case Study enhanced CT sean.
2) lnitial treatment should include urine alkalinizing agents.
A 28-week pregnant patient is brought to the emergency de- 3) Plain x-ray film will show typical staghorn calculus.
partment with a sudden onset of severe left flank pain, irradia- 4) The first step in treatment is administration of analgesic agents.
ted to her groin. She doesn't report dysuria, even in the previous
days. She also presents nausea and vomiting. She is afebrile. No
similar previous episodes were recorded. Which of the following A 34-year-old woman is hospitalized with complaints of right
clauses is the most appropriated? flank an iliac fossa pain that commenced seven hours ago. On-
set was sudden, and the pain is slightly irradiated to her genita-
1) The first test to perform is a plain x-ray film, since it'II enable us lia. She also presented with sweating, nausea, vomits and chills.
to see a lithiasis, if present. Her physical examination reveals a body temperature 36.SºC
2) lt would be reasonable to think that the patient's symptoms are and blood pressure 120/75mmHg. A plain abdominal x-ray film
consistent with a lithiasic reno-ureteral colic, since it's the most shows a calculus. After analgesic treatment, the problem appa-
common urinary complication in pregnant patients. rently resolved so she was discharged; 1O days later, she comes
3) The first diagnostic test performed is a contrast-enhanced CT to your outpatient clinic with the results of an intravenous uro-
sean, the current gold standard for diagnosis of urinary lithiasis. graphy that reveals delayed elimination of contrast throughout
4) Ultrasound may be used to analyze whether a urinary deriva- the right kidney with a ureteric grade II proximal dilation. A 6mm
tion device would be indicated. calculus is also observed at the end of the dilated segment. La-
boratory studies do not show any abnormal parameters. Which
of the following is the recommended attitude in this case?
A 34-year-old woman is admitted to our emergency de-
partment with a history of right flank and right iliac fossa pain 1) An emergent urology referral must be performed.
for the least seven hours. The pain onset was sudden and she 2) Perform a scheduled ureterolithotomy.
also presented sweating, nausea, vomiting and chills. Physical 3) Conservative management may be tried first, by administering
examination reveals a temperature 36.SºC and blood pressure propionic acid.
of 120/75mmHg. Which of the following statements this pa- 4) The best treatment in this case may be extracorporeal shock
tient is correct? wave lithotripsy (EWSL).

03 · Urolitiasis
ERRNVPHGLFRVRUJ
Urología 1 03
A patient presents to the emergency department with a history 1) Hematuria.
of sudden pain after receiving extracorporeal shock wave litho- 2) Steinstrasse.
tripsy to treat a ureteric calculus. What is the most frequent com- 3) Hypertension.
plication ofthis procedure? 4) Renal hematoma.

ERRNVPHGLFRVRUJ
_UrDlogía

Tumores renales

Es fundamental conocer la Asimismo, existe una incidencia aumentada en el riñón poliquístico, en


11 1, ! 1 rJ 1 ,~ 1, ! ll N estirpe histopatológica más la enfermedad quística adquirida de la insuficiencia renal crónica y en los
ENARM frecuente y reconocer el
síndrome de Stauffer.
riñones malformados, como el riñón "en herradura".

Procede de las células del túbulo contorneado proximal, y microscópi-


camente predominan las células claras sobre las granulares y fusiformes.

4.1. Carcinoma de células renales


(adenocarcinoma renal, hipernefroma) La esclerosis tuberosa y la enfermedad Von Hippel-
Lindau se asocian también a otro tumor renal, el an-
giomiolipoma.
Es el tumor sólido renal más frecuente (90%) (Figura 1). Es un tumor fun-
damentalmente de la edad adulta, con mayor incidencia entre los 40 y 60
años, con predominio en el varón 3:2 a excepción de la variedad cromó- Presentación
foba, típica de las mujeres.
La tríada clásica: hematuria, dolor y masa en el flanco ocurre únicamente
en el 10% de los casos y, cuando se presenta así, generalmente se trata de
una enfermedad avanzada.

Actualmente la mayoría de las neoplasias renales son descubiertas inci-


dentalmente en pruebas de imagen de rutina por otra causa.

La anomalía más frecuente es la hematuria macroscópica o microscópi-


ca (60%). Otros hallazgos frecuentes son dolor (40%), pérdida de peso
(30%), anemia (40%), masa en flanco (24%), HTA (20%), hipercalcemia
(6%), eritrocitosis (3%). El 20% de los pacientes presentan como cuadro
paraneoplásico alteración de las enzimas hepáticas sin evidencia de me-
tástasis (síndrome de Stauffer). El síndrome paraneoplásico mas común
es el aumento de VSG.

Ocasionalmente, el adenocarcinoma renal puede producir hormonas


productoras de síndromes clínicos según la sustancia secretada . Entre
estas se encuentran péptidos PTH-/ike, prostaglandinas, prolactina, reni-
na, gonadotropinas o corticoides. La invasión de la vena renal principal
en el lado izquierdo puede ocasionar la aparición de un varicocele de
Figura 1. Carcinoma de células renales forma repentina, que no disminuye en decúbito.

Entre los factores de riesgo que se han implicado se encuentran el humo


del tabaco, el cadmio, la obesidad y la HTA. Existen formas familiares que
La producción de péptidos puede aparecer en el hi-
suelen ser múltiples y bilaterales, como en la enfermedad de Von Hippel- pernefroma, pero es más típica de carcinomas epider-
Lindau y, en menor medida, la esclerosis tuberosa. Se han identificado moides (pulmón, esófago, etcétera) .
alteraciones cromosómicas que implican al cromosoma 3.

ERRNVPHGLFRVRUJ
Urología 1 04
Diagnóstico
La ecografía es la primera prueba complementaria que debe realizarse,
para identificar masas solidas de quísticas y normar la conducta del pro-
tocolo de estudio a seguir. De esta forma se diagnostican la mayoría de
las masas renales en la actualidad (Figura 2).

Figura 4. RM de tumor renal con trombo en venas renal y cava

Aunque la urografía intravenosa (UIV ) anteriormente se consideraba la


base del diagnóstico por imagen en urología, en el caso del adenocarci-
noma renal proporciona pocos datos y de forma indirecta, como puede
ser la distorsión del sistema colector, su ocupación o la anulación funcio-
nal del riñón. En las placas tomográficas de la UIV puede observarse la
presencia de una masa o una alteración del contorno renal.
Figura 2. Ecografía de quistes renales simples
La arteriografía renal, exploración obligada hace años, ha quedado rele-
La realización de punción-aspiración con aguja fina (PAAF) de una masa gada a los casos dudosos, riñones únicos y otras situaciones en las que
renal para su filiación es una exploración agresiva que, debido a su baja se plantea tratamiento quirúrgico conservador. El patrón arteriográfico
sensibilidad, no se justifica actualmente, excepto en casos excepciona- característico incluye neovascularización tumoral, lagos venosos, fístulas
les. arteriovenosas y vasos capsulares.

La tomografía computarizada (TC) es el mejor método aislado para eva- El estudio de extensión, si se sospechan metástasis, se completará rea-
luar una masa renal, proporcionando información precisa sobre metás- lizando radiografía de tórax, las nuevas guias incluso justifican la tomo-
tasis ganglionares (80%) y afectación de órganos adyacentes (Figura grafía de tórax, analítica hepática completa y, en algunos casos dudosos
3) . serie ósea metastasica.

El procedimiento diagnóstico ante la presencia de masas renales se pue-


de observar en la Figura 5.

· Examen físico
· Análisis de orina
!
Masa renal descubierta accidentalmente

EiiiiliiiM
/ ~
-----
t
Observar

Masasóllda
Figura 3. TC de masa renal izquierda o quiste compltcado

La RM, aunque no se emplea de manera rutinaria en este tipo de pacien-


Nefrectomía No complicado: Complicado:
tes, sí se utiliza como método de diagnóstico básico en sujetos en los observación . Nefrectomía simple
radica l
que se sospecha afectación trombótica tumoral de la vena renal o cava o parcial · Nefrectomía parcial
(angioresonancia) (Figura 4). · Embolización
Figura 5. Algoritmo diagnóstico de las masas renales

ERRNVPHGLFRVRUJ
Manual CTO de Medicina y Cirugía, 3.ª edición

Estadificación La linfadenectomía regional no mejora la supervivencia y únicamente


tiene, por tanto, validez en la estadificación, por lo que no se realiza siste-
Actualmente se utiliza el sistema TNM 2009 para estadificar la enferme- máticamente. Este se establece sobre la base de los hallazgos quirúrgicos
dad. Y la escala de Fuhrman para valorar el grado: y anatomopatológicos.
T: Tumor Primario
TO: Sin evidencia de Tumor primario Otras formas de tratamiento carecen de eficacia. Tanto la quimioterapia
T1: Tumor con diámetro$ 7 cm, limitado al riñón. como la radioterapia ofrecen resultados pobres. En el caso de enfermedad
T1 a: Tumor con diámetro$ 4 cm metastásica, las opciones son múltiples, pero ninguna satisfactoria. Aun-
T1 b: Tumor con diámetro >4cm pero$ 7 cm. que se ha descrito la regresión de las lesiones metastásicas tras la nefrec-
tomía, esto ocurre únicamente en un 1%, y generalmente de forma transi-
T2: Tumor con diámetro> 7 cm, limitado al riñón. toria, por lo que no se justifica, salvo de forma paliativa por otros motivos.
T2a: Tumor con diámetro> 7cm pero $ 1Ocm
T2b: Tumor con diámetro> 1Ocm, limitado a riñón. La inmunoterapia con interferones, interleucina, linfocitos killer activados,
y ciertas combinaciones de quimioterapia (mTOR) con inmunoterapia,
T3: Tumor que se extiende dentro de venas mayores o tejido perin- son alternativas para la enfermedad metastásica, pero en ninguna de ellas
efrico, pero no sobrepasa Gerota ni glándula suprarrenal. se obtienen tasas de respuesta superiores al 15%. Actualmente se utilizan
T3a: Tumor se extiende dentro de vena renal o invade seno renal de preferencia: antitirosincinasas en primera línea, antiangiogénicos en
pero no sobrepasa Gerota. segunda línea. Las situaciones que favorecen la respuesta al tratamiento
T3b: Tumor se extiende dentro de vena cava por debajo del inmunoterapéutico son: presencia de metástasis pulmonares exclusiva-
diafragma. mente, buen estado general, y que se haya realizado la nefrectomía antes
T3c: Tumor se extiende dentro de vena cava por arriba del dia- del descubrimiento de las masas pulmonares.
fragma.
En la actualidad se investiga sobre autovacunas elaboradas con linfocitos
T4: Tumor que invade mas alla de Gerota o invade glandula supra- peritumorales que parecen ofrecer resultados alentadores.
rrenal.
N: Ganglios regionales.
NO: sin ganglios invadidos. 4.2. Otros tumores
N1: Metástasis en uno o más ganglios linfáticos.
M: Metástasis a distancia.
MO: Sin metástasis a distancia. Tumor de Wilms (véase Sección de Pediatría).
M1: Con metástasis a distancia. Tumores renales metastásicos. Pueden encontrarse metástasis en
el riñón de tumores de pulmón (la más frecuente), mama, melano-
Tratamiento mas e infiltración por linfoma.
Tumores benignos:
Una vez estudiado el tumor y descartada la presencia de metástasis, Adenomas corticales: son los tumores más frecuentes del adul-
tanto viscerales como ganglionares, el tratamiento de elección es la to, aunque indistinguibles clínicamente del adenocarcinoma,
nefrectomía , ya sea parcial o radical, esta última incluyendo la fascia por lo que se tratan como tales. El criterio clásico de tamaño (3
de Gerota y la glándula suprarrenal, aunque si esta no está invadida cm) para su diagnóstico diferencial no es válido en la actualidad.
macroscópicamente se puede preservar. No se realiza biopsia percu- Angiomiolipomas: se asocian a la esclerosis tuberosa en un 50%.
tánea previa, ya que en sospecha clínica de malignidad, más del 90% Compuestos de una proporción variable de grasa, vasos y fibras
se confirma al analizar la pieza quirúrgica. musculares. Cuando son grandes (mayores de 4 cm), pueden oca-
sionar un síndrome de Wünderlich por sangrado retroperitoneal.
La cirugía conservadora o parcial, está indicada en los tumores bilaterales, Cuando se asocian a esclerosis tuberosa, suelen ser múltiples y
en aquellos que aparecen sobre riñón único, o sujetos con nefropatías bilaterales, por lo que deben tratarse de forma conservadora.
médicas, en los que la pérdida de masa nefronal obligaría a diálisis. Oncocitoma: considerado benigno, aunque en algunos se
han detectado metástasis. Hay criterios radiológicos para dis-
Además de en estas indicaciones imperativas, actualmente es el nuevo tinguirlo del adenocarcinoma, pero en la mayoría de los casos,
patrón de referencia en el tratamiento de tumores pequeños (menos de ni estos ni la citología o la biopsia, ofrecen garantías suficien-
4 cm), bien delimitados y sin afectación de la grasa perirrenal. En estos tes de su benignidad, por lo que tienden a ser tratados me-
pacientes seleccionados parece que la supervivencia y la tasa de recidivas diante nefrectomía.
locales son semejantes a las que se presentan en casos similares tratados Nefroma mesoblástico (hamartoma fetal): es el tumor be-
con nefrectomía radical. nigno más frecuente en recién nacidos y lactantes.

04 · Tumores renales
ERRNVPHGLFRVRUJ
Urología 1 04
" El hipernefroma puede producir multitud de síndromes para-
Ideas clave neoplásicos. Esto puede complicar bastante el diagnóstico, de
ahí el sobrenombre de"tumor del internista''.
" El tumor más frecuente de células renales es el de células claras.
" La elevación de las transaminasas sin afectación hepática es tí-
" El contexto típico es el paciente varón, obeso, fumador de la Sta pica del hipernefroma (síndrome de Stauffer).
o 6ta década de la vida.
" No se debe confundir un quiste simple con un hipernefroma.
" La tríada clásica consiste en hematuria, dolor y masa en flan- Los criterios de quiste simple son: contorno liso, contenido tran -
co, actualmente, lo más habitual es que sea incidentaloma sónico y refuerzo posterior.
(asintomático). Si produce síntomas, el más frecuente es la
hematuria. " La primera prueba de imagen, ante la sospecha de hipernefro-
ma, sería la ecografía .
" Hay que sospechar tumor renal ante un varicocele izquierdo, de
aparición súbita y que no cede con el decúbito. " El tratamiento fundamental del hipernefroma es la extirpación
quirúrgica. La quimioterapia y radioterapia tienen un papel muy
secundario.

3) Angiomiolipoma.
Casos clínicos 4) Nefroblastoma.

Femenino de 64 años de edad, hemotipo A+, antecedente de hi-


pertensión arterial sistémica de 14 años de evolución en trata- Hombre de 45 años tiene carcinoma de células renales extendi-
miento con captopril 50mg cada 12 horas, además de diabetes do. Los niveles de GOT, fosfatasa alcalina, LDH y a-2 globulina
mellitas de 5 años de diagnóstico en tratamiento con insulina son elevados y el tiempo de protrombina alargado. El hígado
NPH 20 unidades por la mañana y 1O por la noche, tabaquismo aparece difusamente agrandado, no existen defectos focales de
a razón de 2 cajetillas al día, inicia padecimiento 6 meses antes infiltración intrahepática. La explicación etiológica más proba-
de su ingreso a nuestra unidad con astenia, adinamia, cefalea, ble para estos hallazgos será:
pérdida de peso a razón de 7kgs y dolor en flanco izquierdo, ul-
trasonido que reporta tumor dependiente de riñón izquierdo de 1) Amiloidosis.
100x95 mm ¿Cuál es la variante histopatológica más frecuente? 2) Secundarismo hepático.
3) Síndrome de Stauffer.
1) Adenocarcinoma. 4) Trombosis tumoral que obstruye la vena hepática.
2) Liposarcoma.

A woman presents to the clinic with a three-month history


Case Study of flank pain. She feels a mass in that area and has lost 12kg
in two months. Her urine is darker than usual and someti-
A 62-year-old patient comes to the hospital with complaints mes she has seen small amounts of blood in her urine. Re-
of difficult micturition, weak voiding and postvoid drib- garding this case, which of the following is correct?
bling. Abdominal kidney ultrasound reveals a 3cm diameter
hypoechoic mass in the right kidney. Posterior acoustic en- 1) The main diagnostic suspicion is renal adenocarcinoma and the
hancement is also observed. Left kidney appears to be nor- prognosis is good since she presents with initial symptoms.
mal. In this case: 2) The main diagnostic suspicion is renal adenocarcinoma and
intravenous urography should be performed to complete the
1) A fine needle aspiration biopsy should not be performed. diagnostic study.
2) An ultrasound-guided fine needle aspiration biopsy should be 3) You should suspect a xanthogranulomatous pyelonephritis. Ne-
performed. phrectomy is indicated per the patient's symptoms.
3) A CT sean guided fine needle aspiration biopsy should be per- 4) The main diagnostic suspicion is renal adenocarcinoma and an
formed. abdominal CT sean should be performed to study the extension
4) A tru-cut biopsy should be performed. of the disease.

An ultrasound shows a right kidney mass that is 3cm in dia- A right renal mass measuring more than 7cm is incidentally
meter and apparently solid. Left kidney seems to be normal. found during a routine ultrasound. Lab tests show: creati-
A CT sean is performed showing that the mass is in the supe- nine 0.95mg/dL, Na: 138mEq/L, K:4.1 mEq/L AST: 293IU/L,
rior half of the right kidney having a fat-like density, sugges- ALT:324 IU/L, PA: 842U/I, TB: 0.95, Hb: 14.2g/dL, white blood
ting an angiomyolipoma. Which of the following is incorrect cell count 9,600 mil/mm3, platelets 180,000mm 3 • in this case:
regarding this case?
1) CT sean will probably disclose metastasic liver lesions.
1) We can put the patient at ease regarding his renal mass. 2) These findings are due to liver compression by the renal mass,
2) A karyotype should be performed to rule out tuberous sclerosis. since it's located within the right kidney.
3) Annual follow-up with CT sean should be performed. 3) The renal function parameters may lead us to considera partial
4) lt could cause Wunderlich syndrome. nephrectomy despite the tumor's size.
4) The hepatotoxic effect of the tumor usually disappears after the
nephrectomy.

ERRNVPHGLFRVRUJ
. Urologia_

Hiperplasia y carcinoma
prostático

' ·. Tanto la hiperplasia prostática benigna como el cáncer de próstata son dos temas fundamentales.
Probablemente el cáncer sea más importante, sobre todo en lo referente al tratamiento.
ENARM Hay que aprenderse muy bien el resumen de la Tabla 11.

5.1. Hiperplasia prostática benigna a través de su forma activa, la dihidrotestosterona, es fundamental,


su papel exacto no ha sido determinado. Las teorías más recientes
La hiperplasia prostática benigna (HPB) afecta en mayor o menor grado abogan por un desequilibrio hormonal de estrógenos/andrógenos,
a la gran mayoría de los varones a partir de la quinta década de la vida, o por la existencia de factores de crecimiento prostáticos con un
alcanzando el 80-95% de la población masculina de 80 años. papel permisivo del ambiente hormonal y alteración de la apoptosis
celular.
La próstata se divide clásicamente en cinco lóbulos (anterior, medio, pos-
terior y dos laterales); aunque estos únicamente se encuentran como No existe evidencia de asociación entre HPB y carcinoma prostático.
tales en la edad fetal. En el adulto se puede interpretar la anatomía de
la próstata dividida en dos partes: una zona periférica, donde se origina Diagnóstico
principalmente el carcinoma, y una zona periuretral o transicional, de la
que procede la HPB (Figura 1). El crecimiento prostático generalmente se produce hacia la zona tran-
sicional alrededor de la uretral, ocasionando obstrucción de esta y
La HPB está compuesta de una proliferación variable de elementos dificultando el vaciamiento vesical (Figura 2). Esto no se manifiesta in-
glandulares, musculares y del estroma, que en su crecimiento com- mediatamente, sino que, generalmente, el proceso pasa por una serie de
primen la próstata periférica, formando la llamada cápsula quirúrgi- etapas que incluyen una fase de compensación, una fase clínica y una de
ca . Su etiopatogenia no está clara; aunque el estímulo androgénico descompensación.

Lóbulo anterior
Estroma fibromuscular anterior

Zona de transición

lateral

Glándulas
suburetrales
Figura 1. Anatomía de la próstata

ERRNVPHGLFRVRUJ
Urología 1 05
Fase de descompensación (Tabla 1 ). Se produce un vencimien-
to del detrusor vesical, que es incapaz de vencer la presión uretral,
aumentando la sintomatología anterior y pudiendo presentarse
retención urinaria. Ocasionalmente puede aparecer dilatación
ureteral bilateral con deterioro de la función renal. Esto se debe a
uropatía obstructiva infravesical con pérdida del mecanismo anti-
rreflujo.

Síntomas
Síntomas Síntomas de llenado
obstructivos
postmiccionales o "irritativos"
o de vaciado
Dificultad de inicio Sensación Urgencia miccional
miccional de vaciado Nocturia
Disminución incompleto
Polaquiuria diurna
del calibre y fuerza Goteo
Dolor suprapúbico
del chorro miccional postmiccional
Figura 2. Ecografía de hiperplasia prostática con crecim iento del Incontinencia
Micción
lóbulo medio intravesical por urgencia
intermitente
miccional
y prolongada
Fase de compensación. El crecimiento prostático ocasiona un au- Retención de orina
mento de la presión uretral durante el vaciado que es compensado e incontinencia
por una mayor actividad contráctil del detrusor que se hipertrofia, por rebosamiento

encontrando presiones vesicales más elevadas. En esta fase, la clínica Tabla 1. Síntomas HPB
puede ser mínima o inexistente.
Fase clínica (Figura 3). La elongación de las fibras musculares por Pueden producirse también otro tipo de síntomas denominados de lle-
encima de un límite condiciona pérdida de capacidad contráctil. En nado o "irritativos'; que son debidos a la alteración funcional vesical, y
este momento aparece retraso del inicio de la micción, disminución cuya resolución es más difícil tras la desaparición de la obstrucción. En-
del calibre y de la fuerza del chorro miccional y alargamiento del va- tre estos síntomas se incluyen polaquiuria, tenesmo, nicturia y urgencia
ciado (lo que en conjunto se denomina síntomas obstructivos). El va- miccional. La HPB es la causa más frecuente de obstrucción del tracto
ciado suele ser incompleto, dando lugar a un residuo posmiccional. urinario inferior en el varón.

La mejor herramienta para valorar los síntomas urinarios bajos es la


escala internacional para síntomas prostáticos (IPSS). (Anexo 1) SI bien
no es diagnostico, es una herramienta clave para valorar la respuesta
al tratamiento y calidad de vida de los pacientes

En la evaluación del síndrome prostático, el tacto rectal continúa sien-


do la exploración fundamental, sobre todo para diferenciarlo del car-
cinoma, ya que no es infrecuente que ambas entidades coexistan. La
clínica es lo más importante para valorar la indicación de tratamiento
de la HPB, ya que no existe correlación entre el tamaño prostático y
el grado de obstrucción. Cualquier zona sospechosa al tacto debe ser
biopsiada.

La medición del flujo máximo en una uroflujometria miccional es tam-


bién importante, considerándose normal cuando es mayor de 15 ml/s
y claramente patológico si es menor de 1O ml/s. El estudio puede com-
pletarse con una ecografía que permita evaluar si existe afectación del
tracto urinario superior, residuo posmiccional, litiasis vesical u otra pato-
logía asociada. El uso del PSA en la HPB únicamente está indicado para
descartar la presencia de carcinoma en la próstata, ya que no sirve para
diagnosticar HPB.

Tratamiento
Dentro de las posibilidades terapéuticas, la cirugía continúa siendo el úni-
co tratamiento definitivo para la HPB. Esta puede ser endoscópica (RTUp:
Urografía intravenosa mostrando impronta prostática en vejiga (vejiga "en resección transuretral prostática) o abierta (adenomectomía prostática)
montera") con uréteres en anzuelo (Figura 4), dependiendo del tamaño del adenoma. En el 10% de las pie-
Figura 3. Hiperplasia prostática en fase clínica zas obtenidas se encontrarán focos de adenocarcinoma incidental.

ERRNVPHGLFRVRUJ
M anual CTO de Medicina y Cirugía, 3.ª edic ión

5.2. Carcinoma prostático

El adenocarcinoma prostático es el tumor maligno más frecuente del apa-


rato genitourinario masculino y el segundo en frecuencia general, después
del pulmonar. Sin embargo, si se incluyesen los carcinomas incidentales y
los encontrados en autopsia, supera al pulmonar en prevalencia.

La hormonodependencia del cáncer prostático parece indicar el papel


de los andrógenos en su etiología o patogenia. La relación de factores
genéticos, ambientales o infecciosos no ha quedado suficientemente es-
tablecida.

El 95% de los carcinomas prostáticos son adenocarcinomas originados


en la zona periférica de la próstata. Los carcinomas ductales se originan
en los conductos prostáticos en lugar de los acinos, e histológicamente
Figura 4. Adenomectomía prostática pueden corresponder a carcinomas transicionales, escamosos, endome-
trioides o mixtos. Más raros son los carcinosarcomas (menos del 1%).
Se debe tener en cuenta que en la cirugía de la HPB no se extirpa la cáp-
sula quirúrgica, que está constituida por las glándulas prostáticas perifé- El adenocarcinoma prostático, con frecuencia, es multifocal y presenta po-
ricas comprimidas por el adenoma, y es el principal origen del carcinoma blaciones en distinto grado de diferenciación. En esta heterogeneidad se
prostático, por lo que la intervención quirúrgica no protege del desarrollo basa la clasificación de Gleason, que asigna una puntuación de 1 a 5, según
de este proceso. el patrón histológico de cada una de las dos poblaciones más representa-
tivas de la masa, sumando ambas puntuaciones para obtener un resultado
Los tratamientos no quirúrgicos son fitoterapia (poco efectivo); antagonis- final de 2 a 1O. Esta escala de Gleason se corresponde con el pronóstico de
tas a-adrenérgicos (tamsulosina, silodosina, doxazosina, etc.) que relajan la la enfermedad, independientemente del estadio. Para la estadificación se
musculatura del cuello vesical y de la uretra; inhibidores de la 5-a-reductasa emplea principalmente la clasificación TNM (Tabla 2 y Figura 5).
(finasteride, dutasteride) reducen el tamaño prostático. En casos avanzados
con próstata mayor de 40 cm 3 se recomienda iniciar con terapia combina-
da de estos fármacos. Para los síntomas de llenado además se pueden aso-
ciar antimuscarínicos (solifenacina, oxibutininia, tolterodina ... ) o agonista
a-3 (mirabegron) que son fármacos utilizados para vejiga hiperactiva T1: tumor inaparente clínicamente (no palpable ni visible
por técnicas de imagen)
- Tl a: hallado incidentalmente. Afectación menor del 5% del tejido
resecado
- Tl b: hallado incidentalmente. Afectación mayor del 5% del tejido
La fi nasterida también es útil para la alopecia andro-
resecado
génica, donde se emplea en dosis mucho menores.
- Tl e: Tu mor identificado por punción-biopsia por aumento del PSA

T2: tumor confinado a la próstata (incluye la invasión de la cápsula


prostática sin exteriorización del tumor hacia el tejido adiposo
Como inconvenientes principales de los inhibidores de la Sa-reductasa, peri prostático)
se encuentran: impotencia, reducción del PSA en torno al 50% (dificultan- - T2a: menos del 50% de un lóbulo
do el diagnóstico del carcinoma, si lo hubiese) y que tarda una media de - T2b: más del 50% de un lóbulo
cuatro meses en hacer efecto. - T2c: dos lóbulos

T3: extensión del tumor por fuera de la cápsula


De los a-bloqueantes los inconvenientes principales son la hipotensión - T3a: extensión transcapsular (sea unilateral o bilateral)
(más frecuente en los fármacos antiguos) y la eyaculación retrógrada. - T3b: invasión de la(s) vesícula(s) seminal(es)

T4: tumor fijo o invade órganos adyacentes distintos a las vesículas


En cuanto a las indicaciones de tratamiento quirúrgico, globalmente, sólo seminales (cuello vesical, esfínter externo, recto, músculo elevador
un 10% de los pacientes prostáticos precisará cirugía. La intensidad de las o pared pelviana)
manifestaciones clínicas subjetivas y la mala respuesta al tratamiento médico N: define la afectación ganglionar
pueden constituir la indicación para la intervención. Entre las causas "objeti- NX: no se pueden estudiar los ganglios regionales
vas" que suponen indicación absoluta de tratamiento quirúrgico quirúrgico · NO: no metástasis ganglionares
y que además son las compliaciones mismas que causa la enfermedad se · N1: metástasis a ganglios regionales
encuentran:: M: define las metástasis
Retención urinaria reiterada. MO: no metástasis
Hidronefrosis retrógrada (lesión del parénquima renal por obstruc- M1: metástasis a distancia
ción infravesical). - M1a: ganglios linfáticos no regionales
Infección urinaria de repetición. - Ml b: hueso
- M1e: otras localizaciones
Litiasis vesical.
Hematuria de repetición. Tabla 2. Estadificación del carcinoma de próstata

05 · Hiper plasia y carcinoma prostático


ERRNVPHGLFRVRUJ
Urología 1 05
T: 1Umor primario clínica TN

T2a T2b
TX No puede evaluar el tumor
TO No existen signos de tumor primario

Tla Tlb
''
''
''
11
''
ii
''
: 1
< 5% ! ! > 5%
T2 Tumor limitado a la próstata T3 Tumor que se extiende a través
T1 Tumor no evidente clínicamente, no palpable de la cápsula prostática:
o a la cápsula, sin sobrepasarla:
ni visible mediante técnicas de imagen: · T3a Extensión extracapsular (unilateral
· T2a Menos del 50% de un lóbulo
· T1 a Extensión menor o igual al 5% del tejido o bilateral)
· T2b Más del SO% de un lóbulo
resecado · T3b Tumor invade la vesícula seminal
· T2c Dos lóbulos
· T1 b Extensión mayor del 5% del tejido
resecado
· T1cTumor identificado mediante punción
biópsica (consecuencia de un PSA elevado)

T4 Tumor fijo o que invade estructuras adyacentes diferentes a las vesículas seminales NX No se pueden evaluar los ganglios linfáticos regionales
NO No hay metástasis ganglionares regionales
N 1 Metástasis en ganglios linfáticos regionales

Figura 5. Estadificación del adenocarcinoma de próstata

Clínica EE.UU. o Europa, donde más del 70% de estos tumores se diagnostican en
fases tempranas de la enfermedad.
El carcinoma prostático es una enfermedad más frecuente en ancianos,
y la mayoría de ellos se diagnostica por encima de los 60 años. Clín ica- Marcadores tumorales
mente puede producir síntomas obstructivos del tracto urinario inferior
superponibles a los de la HPB. A estos puede añadirse la hematuria. El Se dispone fundame nta lmente de dos ma rcadores tumorales. La fosfa-
25% de los pacientes que refieren retención urinaria aguda presentan un tasa ácida prostática (FAP) se emplea en clínica desde hace décadas, es
carcinoma prostático. Aproximadamente un 25% de los pacientes pre- un marcador específico, pero su elevación suele indicar extensión extra-
sentan metástasis en el momento del diagnóstico; estas pueden produ- prostática, por lo que no resulta útil en el diagnóstico precoz. El antígeno
cir manifestaciones como dolor óseo, compresión medular, mieloptisis prostático específi co (PSA) es rea lmente un ma rcador de tejido prostático
o coagulopatía. Afortunadamente, estos casos se encuentran en claro cuyos niveles suelen encontra rse más elevados en el cáncer, pero es ines-
descenso gracias a la incorporación del PSA (prostate-specific antigen-an- pecífico y ta mbién están elevados a consecuencia de patología benigna
tígeno prostático específico), fac ilitando el diagnóstico de la enfermedad (infecciones, sondajes, HPB, etc.). Por este motivo se ha intentado aumen-
en estadios tempranos y comúnmente asintomáticos. tar su especificidad para cáncer con otros parámetros (densidad de PSA,
índice PSNedad, velocidad de ca mbio del PSA, PSA libre, calicreinas),
Diagnóstico aunque aún no ha quedado establecida la ventaja de estos sobre el PSA
aislado, sin emba rgo, los resu ltados pa recen alentadores.
Tacto rectal
Si el PSA es menor de 4 ng/ml, es poco probable que se encuentre un
Continúa siendo el método fundamental de cribado. Son accesibles al cáncer de próstata. Si es mayor de 1O, las probabilidades aumentan, lo
tacto rectal todos los estadios excepto el Tl, que por definición es un que aconsejaría una biopsia de próstata ecodirigida. Si está entre 4 y 1O,
hallazgo. Ca racterísticamente, el carcinoma es duro, nodular e irregu lar. se pueden utilizar los parámetros antes mencionados para valorar la ne-
En general, se aconseja un tacto recta l y un PSA anual a todos los varones cesidad de biopsia.
por encima de 50 años y hasta los 70 años, aunque, de momento, la OMS
no aconseja la realización de cribado poblacional sistemático. No obstan- Pruebas de imagen
te, en México en la actualidad el 75% de los cánceres de próstata se diag-
nostica ya en estado avanzado, por lo que sería conveniente plantearse la La ecografía transrecta l (ETR) (Figura 6) es el método de imagen más
real ización de un screening para trata r de aproximar estas cifras a las de útil para la estadificación local, pud iendo ofrecer información importante

ERRNVPHGLFRVRUJ
Manual CTO de Medicina y Cirugía, 3.ª edición

sobre la afectación capsu lar, de vesículas seminales, cuello vesical o recto. ciente en quien se sospeche metástasis Gleason >8, PSA >20 = Riesgo
Aunque no existe un patrón ca racterístico, suele aparecer como nódulos Alto). Antes de plantearse el tratamiento curativo, en ciertos pacientes
hipoecogénicos. La ecografía transrectal ofrece, además, la posibilidad de con altas probabilidades de encontrarse el cáncer extendido, se debe
dirigir la biopsia hacia las zonas sospechosas. efectuar una gammagrafía previa para confirmar la no existencia de me-
tástasis óseas o una TC para descartar metástasis ganglionares.
La ecografía abdominal no tiene gran valor en la detección del carcinoma
prostático. La TC y la RM tienen su principal papel en la estadificación
ganglionar y la valoración de metástasis a distancia. Las primeras metás-
tasis deben buscarse a nivel de los ganglios linfáticos de las cadenas ob-
turatrices e ilíacas.

Actualmente existen escalas para determinar que pacientes son de alto


riesgo para enfermedad extraprostatica, como la calsificación según ries-
go D'amico, que considera estadio TNM, Niveles de PSA y resultado histo-
patológico de Gleason de Biopsia prostática.

Figura 7. Gamagrama óseo positivo a metástasis óseas por Ca próstata.


Las fechas blancas muestran los focos de actividad osteoclastica .

Las metástasis del cáncer de próstata son osteoblás-


ticas es decir, forman, su actividad es mejor valorada
en un gamagrama óseo.
Sitio de metástasis más frecuente: columna.

Biopsia prostática

Debe realizarse para la confirmación del diagnóstico. Puede efectuar-


se vía transrectal o transperineal, guiada por el tacto recta l o bien
guiada por la ecografía transrecta l, lo que añade efectividad a la prue-
ba. La realización de la biopsia está indicada siempre que exista una
anomalía del tacto rectal, elevación de los marcadores tumorales o
alteración en las pruebas de imagen. La punción-aspiración con agu-
ja fina (PAAF) es una alternativa con menores complicaciones, pero
con el inconveniente de que no puede evaluar el grado histológico
(Gleason).

Son indicaciones de biopsia prostática el tacto rectal


sospechoso, la presencia de un nódulo ecográfico y
un PSA > 4 (variable la cifra según criterios).

(A) ETR corte transversal; (B) ETR corte longitudinal; (C) Adenocarcinoma,
nódulo hipoecoico en lóbulo derecho
Figura 6. Ecografía transrectal (ETR) de adenocarcinoma prostático Tratamiento

Gammagrafía ósea Opciones terapéuticas

Se utiliza para la detección de metástasis óseas, tiene mayor sensibilidad Prostatectomía radical. Los pacientes candidatos deben ser indi-
que la radiología convencional (Figura 7), y debe realizarse en todo pa- viduos con una esperanza de vida superior a 1O años. Como com-

05 · Hiperplasia y carcinoma prostático


ERRNVPHGLFRVRUJ
Urología 1 05
plicaciones, se puede encontrar incontinencia (2-57%), estenosis Otros: se están diseñando nuevas moléculas que bloquean losan-
anastomótica (10%), impotencia (50%) o incluso la muerte(< 5%). drógenos a distintos niveles como la abiraterona y la enzalutamida,
En líneas generales, suele ir acompañada de linfadenectomía íleo- utilizándose cuando el tumor se hace resistente a la hormonotera-
obturatriz. pia convencional.
Radioterapia. Como tratamiento curativo, los resultados en esta-
dios localizados se acercan a los de la cirugía. La diarrea crónica, la Tratamiento por estadios
proctitis, la cistitis rádica y las fístulas urinarias son complicaciones
del tratamiento, así como la incontinencia y la impotencia a partir Estadio T1 a. Tienen una mortal idad por la enfermedad del 2% a los
de los dos años de tratamiento. 1Oaños, por lo que no precisan tratamiento, salvo quizá los pacientes
Se ha empleado también radioterapia intersticial (braquiterapia) jóvenes (menores de 60 años) con una elevada esperanza de vida.
con implantación de yodo-123 (1-123), oro-198 (Au-198), paladio Estadio Tl b-T1 c. Puede ofrecerse Vigilancia activa en pacientes
e iridio. Su indicación queda limitada a tumores pequeños de es- con bajo riesgo, sin embargo el tratamiento de referencia es la
tadio Tl o T2, y sus resultados son similares a los de la cirugía. En prostatectomia radical, si hay contraind icación para cirugía se pue-
caso de compresión medular o dolor por metástasis óseas, la ra- de ofrecer Rt, si no son aptos para ninguna de las anteriores se
dioterapia sobre la metástasis puede conseguir el control local de ofrece terapia hormonal.
la enfermedad. Estadio T2a. Es la indicación más clara de prostatectomía radical.
Vigilancia activa: en los tumores de bajo riesgo, es posible realizar La radioterapia o braquiterapia se reservaría para pacientes de ries-
vigilancia con controles periódicos y en caso de progresión realizar go quirúrgico elevado o que no aceptan efectos secundarios atri-
tratamiento con intención curativa. buibles a la cirugía.
Hormonoterapia. El adenocarcinoma prostático está compuesto Estadio T2b y T2c. Un 40% demuestra ser en realidad estadio 3,
por una población heterogénea de células androgenodependien- t ras el análisis de la pieza quirúrgica de prostatectomía radical (in-
tes y androgenoindependientes. La supresión hormonal frena el fraestadificación). La radioterapia externa o braquiterapia también
crecimiento de las primeras, pero no afecta a las androgenoinde- puede ser útil en pacientes de alto riesgo quirúrgico.
pendientes. Se puede conseguir disminuir los niveles de andróge- Estadio T3a. La indicación quirúrgica es dudosa, así como la ra-
nos circulares por distintos métodos: dioterapia local, por lo que solamente se propondría a sujetos
Castración quirúrgica. Es el método aislado más eficiente, j óvenes, aun a costa de obtener malos resultados. Generalmente
con la ventaja de que elimina la necesidad de medicación son tratados como el grupo siguiente.
permanente. Por su rapidez en el efecto supresor hormona l, Estadio T3b, T4, N+, M+. Varón añoso con mal estado general.
también está indicada en las compresiones medulares por El tratamiento hormonal es la opción indicada. Puede ser preci-
metástasis. so el uso de radioterapia paliativa sobre la metástasis en caso de
Estrógenos (dietilestilbestrol). Inhibe la secreción de LH. do lor. En pacientes asintomáticos incluso se puede realizar una
Actualmente este método se ha abandonado debido al alto actitud expectante y tratar los síntomas que aparezcan confor-
riesgo cardiovascular que conlleva . me la evolu ción de la enfermedad .
Progestágenos. Inhiben la secreción de LH y actúan como
antiandrógenos, al unirse a los receptores de la dihidrotestos- Recidiva tumoral posterior a tratamiento
terona. Es preciso añadir estrógenos para evitar el fenómeno con intención curativa
de escape, que se produce tras varios meses de tratamiento.
No son de uso habitual.
Agonistas LHRH. Aunque inicialmente ocasionan un aumen- Tras la rea lización de prostatectomía radical, los pacientes son moni-
to de los niveles de testosterona, posteriormente suprimen la torizados generalmente con peticiones de PSA.
secreción de LH y de andrógenos. La elevación transitoria de
los andrógenos puede empeorar el cuadro clínico, principal- Cuando las cifras de PSA tras prostatectomía radical son supPrim2s a
mente si existe compromiso medular por metástasis óseas. 0,4 ng/ml, se considera recidiva bioquímica y debe hacer sospechár
Esta elevación (flare-up) se debe suprimir med iante la adminis- la existencia de metástasis a distancia, o bien la existencia de rec,diva
tración de antiandrógenos, previamente a la introducción de a escala local.
inhibidor de la LHRH.
Antiandrógenos (bicalutamida, flutamida, acetato de ci- Tras la realización de radioterapia como tratamiento de cáncer de
proterona). Compiten con el receptor androgénico. Suelen próstata localizado, los descensos de PSA van siendo paulatinos (a di-
utilizarse con agonistas de la LHRH. El acetato de ciproterona, ferencia de la prostatectomía radical) hasta conseguir un valor nadir,
además de actuar como antiandrógeno, tiene un efecto pro- que es el valor mínimo alcanzado tras el tratamiento y que se conside-
gestágeno, por lo que actúa a nivel central, disminuyendo los rará referencia para el seguimiento posterior. Generalmente se toma
pulsos de LH. el nivel de PSA posterior a 1O vidas medias del mismo como nadir,
Quimioterapia. No es muy efectiva en el adenocarcinoma osea 32 días. (vida media de PSA 3.2 días).
prostático. Se han realizado tratamientos con fármacos, que
son una mezcla de un estrógeno y una mostaza nitrogenada Existen diferentes criterios para considerar el diagnóstico de recidiva
(fosfato de estramustina), pero los estudios son contradicto- bioquímica tras tratamiento con radioterapia: cuando se constata la
rios y las respuestas pobres. Existen estudios prometedores existencia de tres elevaciones sucesivas a partir del valor nadir, cuan-
con el empleo de docetaxel en pacientes con tumores hor- do se evidencian niveles nadir+2 (criterio de la ASCO), o con valores
monorresistentes. nadir+3 (criterio de Philadelphia).

ERRNVPHGLFRVRUJ
Manual CTO de Medicina y Cirugía, 3.ª edición

Tratamiento de urgencia Se pueden disminuir los niveles de andrógenos mediante castración


quirúrgica urgente, ketoconazol en altas dosis o dietilestilbestrol intra-
La compresión medular por el cáncer prostático no tratado puede venoso.
ser la forma de presentación y constituye una urgencia importante.
El objetivo del tratamiento debe ser la supresión androgénica rápida A continuación se expone una tabla que resume tanto las características
o la descompresión medular mediante laminectomía quirúrgica o ra- de la hiperplasia prostática benigna (HPB) como las del adenocarcinoma
dioterapia. prostático (Tabla 3).

HPB Adenocarcinoma prostático


Localización Zona transicional Periférica
Clínica Fases: Mayoría asintomáticos:
Compensación Hasta 25% Síndrome prostático
Clínica Hasta 25% Retención aguda
Descompensación Hasta 25% Metástasis
Diagnóstico Tacto rectal
diferencial Ecografía transrectal (estadificación local)
Gammagrafía osea (metástasis óseas)
PSA (muy sensible, poco específico). Descarta cáncer prostático, pero no diagnostica HPB
Fosfatasa ácida (muy específica, poco sensible)
Biopsia (confirmación)
Tratamiento Fitoterapia Localizado: prostatectomía radical más linfadenectomía
Fármacos: finasterida, u-bloqueadores bilateral, radioterapia
Cirugía: adenomectomía: endoscópica o abierta Avanzado: castración: quirúrgica (elección), farmacológica

Tabla 3. Tabla-resumen de las características de la HPB y del adenocarcinoma prostático

AborclaJC cl 1a9nost1co del paciente con sospechd clP HPB


1 50 anosl

r--..~ ,1..·_,·-- t i, • , !, "(''\.?i


No <,. '
. ·.
,::;.,

~
.
l

• ., .~ ' - ,;; ,,..


-~
_1

~ _.._...

.... •. '""'" ,,.


Tratamiento
sr ~. ~,.
- . especffico
i ~. ··.,-i\'tf
';~,__,_ v~" ~~~

1. Vigilancia activa
2.Medidas
Tratamiento
quirúrgico de HPB
(ver algorltorno tto)

Figura 8. Estadificación del adenocarcinoma de próstata

05 · Hiperplasia y carcinoma prostático


ERRNVPHGLFRVRUJ
Urología 1 05
D1agnost1co y tratdm1ento dl·I CdrllU de prcstc1trl

+
Paciente con sospecha de ca ncer de próstata

Sólo llevar monltoreo


conAPE

+
APE<10mg/ml APE de 10 a 20 mg/ml APE de 20 mglml APE > 20 mg/ml
CiellollS6'J. . . . Glelsonde 7oestldlo Glelson de •• 10 Glalonclt-1 a 10
dllklo'n•T2c c:llnko T2b. T2c lltadlo clnlc:D T3 lltadlodfnlc:oT4
C.dt,..,.. localllldo C.dt¡ñstatalo .......
Bljorlllgo Rlesgolnlllrmedlo ~-~ localmenl9
avenado Alto riesgo
C.depÓIIIMllavanzado
Altorlllgo


Vlgllancia activa
Monltoreo con APE
y biopsias periódicas

Se dlscutira con el pacientl!
continuar con vlgilancla
- •-
Prostatectomla radical
+ hormoterapla
+
Hormonoterapla

o realizar prostatectomia o radioterapia externa +


o radical hormonoterapla
prostatectomia radical o raidoterapia externa u hormonoterapia
o braquiterapia o braquiterapia
o radioterapia externa
__. l
Figura 8. Estadificación del adenocarcinoma de próstata

-
1 Menos de una M~nos de la Aprx. La mitad ~ás de la
Cuestionario IPSS Ninguna vez cada 5 mitad de las d I mitad de las
e as veces
1 veces veces veces
Durante más o menos los últimos 30 días,
¿Cuántas veces ha tenido la sensación de no vaciar o 2 3 4 5
completamente la vejiga al terminar de orinar?
Durante más o menos los últimos 30 días,
¿Cuántas veces ha tenido que volver a orinar a las o 2 3 4 5
dos horas siguientes después de haber orinado?
Durante más o menos los últimos 30 días,
¿Cuántas veces ha notado que, al orinar, paraba y o 2 3 4 5
comenzaba de nuevo varias veces?
Durante más o menos los últimos 30 días,
¿Cuántas veces ha tenido dificultad para o 2 3 4 5
aguantarse las ganas de orinar?
Durante más o menos los últimos 30 días,
¿Cuántas veces ha observado que el flujo de o 2 3 4 5
orina es poco fuerte?
Durante más o menos los últimos 30 días,

-
¿Cuántas veces ha tenido que apretar o hacer o 2 3 4 5
fuerza para comenzar a orinar?
1;0,¡.;:;,;1 1 vez 3 veces 4 veces 5 o más
Durante más o menos los últimos 30 días,
¿Cuántas veces suele tener que levantarse para
orinar desde que se va a la cama por la noche
o 2 3 4 5
hasta que se levanta por la mañana.

-
Tan satisfecho
como
insatisfecho
¿Cómo se sentirla si tuviera qe pasar
el resto de la vida con los síntomas o 2 3 4 5 6

Tabla 4. Escala internacional de síntomas prostáticos (IPSS)

ERRNVPHGLFRVRUJ
Manual CTO de Medicina y Cirugía, 3.ª edición

" El PSA elevado no es diagnóstico de cáncer de próstata. Pue-


Ideas clave de corresponder a una HPB. El diagnóstico definitivo de cáncer
prostático precisa una biopsia.
" La hiperplasia prostática benigna (HPB) suele afectar a la zona
periuretral de la glándula. El cáncer aparece en la zona periférica. " Las metástasis lumbares son típicas del cáncer de próstata, pu-
diendo producir compresión medular.
" La hiperplasia prostática benigna no guarda relación con el cáncer.
" El tacto rectal revela una próstata pétrea e irregular en el cáncer
" Tanto la HPB como el cáncer tienen relación con las hormonas de próstata. Sin embargo, al principio puede no ser palpable, ni
sexuales, y suelen aparecer en varones ancianos. visible en la ecografía (Tl).

" El tratamiento médico de la HPB consiste en a-bloqueadores (re- " La principal complicación quirúrgica del cáncer de próstata es
lajan la musculatura uretral y del cuello vesical), inhibidores de la la impotencia.
Sa-reductasa (disminuye el tamaño glandular) y fitoterapia. Esta
última no ha demostrado utilidad con parámetros objetivos. " Ante un síndrome de compresión medular por cáncer de prós-
tata, nunca se deben emplear análogos de la LHRH únicamente.
" El tratamiento definitivo de la HPB es la cirugía, que puede con- Siempre deben asociarse antiandrógenos.
sistir en resección transuretral o en cirugía abierta, dependien-
do del tamaño prostático. " En el cáncer de próstata, la indicación más clara de prostatecto-
mía radical es el estadio T2a.
" El cáncer de próstata es casi siempre un adenocarcinoma, con
gran frecuencia multifocal. " El tratamiento fundamental del cáncer de próstata diseminado
es la hormonoterapia.
" El cáncer de próstata cada vez se diagnostica con más frecuen-
cia en fase asintomática. Cuando presenta clínica, puede consis-
tir en síntomas urinarios similares a la HPB.

Paciente acude nuevamente a consulta con reporte de patolo-


Casos clínicos gia en donde estatifican Gleason 3+4 = 7, conforme a este ha-
llazgo ¿cual es el tratamiento más indicado en este caso?
Paciente masculino de 65 años de edad, con hipertensión ar-
terial sistémica con tratamiento con losartan, que inicia con 1) Hormonoterapia.
disminución de calibre y fuerza miccional, pujo y tenesmo, al 2) Radioterapia externa + Hormonoterapia.
tacto rectal se encuentra próstata aumentada de tamaño, de 3) Prostatectomía radical + Hormonoterapia.
consistencia dura en ambo lóbulos, superficie nodular y limites 4) Prostactectomía radical.
mal definidos, trae consigo examen de antígeno prostático el
cual es de 4.7, acorde a estos datos ¿cuál es el siguiente examen
que se debe de realizar? Acorde al sitio más frecuente de metástasis en carcinoma pros-
tático ¿cuál es el estudio que debe de realizarse?
1) Tomografía axial computarizada.
2) Gammagrama óseo. 1) TAC de cráneo.
3) No se requieren estudios complementarios debido a que no 2) Biopsia renal.
existen alteraciones en el antígeno prostático. 3) Punción lumbar.
4) Paciente es candidato a realización de biopsia transrectal de 4) Gammagrama óseo.
próstata.

no urinary symptoms or sexual dysfunction. In this particular


Case Study case, which of the following options is correct?

A 65-year-old man is brought to your institution complaining of 1) The next step in the diagnostic workup should be a transrectal
voiding problems. He claims he has difficulty in starting mictu- ultrasound.
rition, weak stream and postvoid dribbling. He presents noctu- 2) The symptoms are not consistent with prostate cancer.
ria twice per night. Laboratory test reveals PSA: 1.37ng/ml. KUB 3) The likelihood of having prostate cancer is approximately 25%
(kidneys, ureters, bladder) ultrasound study shows a normal kid- given his PSA level.
ney and a urinary bladder with a prostate that measu res 45cc. 4) The next step should include a radical prostatectomy.

1) This patient does not need medical treatment right now. A 71-year-old patient presents to his physician with the results
2) A prostate biopsy should be performed. of a blood test showing PSA levels of 15.3 ng/ml. He says he has
3) a-blocker agents should be prescribed. sorne voiding problems, with frequent interruptions and drib-
4) Finasteride treatment is indicated. bling. He often wakes up at night to urinate. He says he has no
hematuria, incontinence or sexual impairment. Digital rectal
A 63-year-old patient comes to the physician beca use he is found examination revea Is an adenomatous grade 111/IV prostate with a
to have a PSA of 5.3 ng/MI on a routine blood analysis. He reports suspicious left-sided nodule. An ultrasound-guided biopsy is per-

05 · Hiperplasia y carcinoma prostático


ERRNVPHGLFRVRUJ
Urología 1 05
formed with a resultant diagnosis of prostate adenocarcinoma A 71-year-old male who has been found to have a PSA of 16ng/
with Gleason score of 8 (4+4). Bone sean shows two lesions that MI undergoes a transrectal biopsy that suggestes prostate can-
probably correspond to metastatic lesions in the lumbar spine cer with a Gleason score of 7 (4+3). lnitial bone sean and abdo-
and scapula. Mark the incorrect sentence about prostate cancer mino-pelvic CT sean do not show any suspicious lesions. The
metastatic lesions: patient is subjected to a radical prostatectomy. Five years later
he complains of right scapular pain and laboratory teste reveals
1) The vertebrae, sternum, pelvis, ribs and femur are the most PSA 6.02ng/MI. A new bone sean shows three suspicious lesions
commonly affected bones. that suggested metastatic lesions, two of them localized in the
2) Osteoblastic and osteoclastic activity increase, but osteoclas- right scapula. In this case, which of the following sentences des-
tic activity is higher and hence osteolytic pattern is more fre- cribes the best action to take?
quently observed in radiographic studies.
3) Pain, inflammation and pathologic fractures are frequent clini- 1) Radiotherapy of the metastatic lesions is indicated.
cal manifestations. 2) Complete hormone-blocking treatment should be initiated.
4) Castration improves symptoms in up to 80% of patients with 3) A wait watchful waiting approach see is recommended, if no
metastatic prostate cancer. pathologic fractures occurred.
4) Curative radiotherapy should be administered.

ERRNVPHGLFRVRUJ
_______Urología _

Carcinomas del tracto urinario

Es fundamental la parte El adenocarcinoma primario vesical es un tumor raro, aunque es el que se

ENARM de tratamiento. ha visto asociado a la extrofia vesical con mayor frecuencia.

No hay que confundir Schistosoma haematobium


con Schistosoma mansoni, que produce hipertensión
portal.
6.1. Carcinoma vesical
Histología e historia natural
El carcinoma vesical es la segunda neoplasia urológica en frecuencia.
Aparece más frecuentemente en varones (2-3:1) y más en población Haciendo referencia al carcinoma de células transicionales, hay que dife-
blanca que negra. Su edad de máxima incidencia se sitúa entre los 60 y renciar tres formas de la enfermedad con comportamiento, pronóstico y
70 años. De ellos, el 90% son carcinomas transicionales, el 8% escamosos tratamiento completamente distinto (Tabla 2 y Figura 1). El 70% de los
y el resto adenocarcinomas. El epitelio urotelial recubre el tracto urinario tumores vesicales se presentan como tumores papilares de crecimiento
desde las papilas caliciales hasta la uretra prostática, ambas inclusive. En principalmente endocavitario y frente de invasión único. Un 10% son sóli-
cualquiera de estos niveles pueden desarrollarse los tumores uroteliales, dos, con invasión tentacular en profundidad y extensión linfática y vascular
correspondiendo la mayoría a la vejiga (más del 90%) y, más raramente, al temprana. El 20% restante son formas mixtas. El primer grupo suele corres-
tracto urinario superior (5%) o la uretra (1 %). pondera tumores superficiales de bajo grado histológico, mientras que los
sólidos, con mayor frecuencia, son tumores infiltrantes de grado histológi-
Entre los factores etiológicos (Tabla 1), se implican las aminas aromáti- co más elevado. La principal característica de los tumores papilares superfi-
cas, presentes en las industrias textiles, químicas y del caucho. El humo ciales es la recurrencia, que ocurre en un 50-75%, según el grado y estadio.
del tabaco es el principal factor de riesgo (50-60% aparecen en fuma- El 25% recurrirán y progresarán en grado y estadio, y únicamente el 15%
dores), aumentando el riesgo a mayor consumo. También pueden jugar acabará desarrollando un tumor infiltrante o metastásico. La mayoría de los
un papel importante los edulcorantes artificiales (sacarina, ciclamato), la tumores infiltrantes se encuentran confinados a la vejiga en el momento
ciclofosfamida, los acetiladores lentos (mayor riesgo) y muchas otras po- del diagnóstico, y sólo un 20-25% presentan extensión ganglionar o metas-
sibles etiologías. La infestación por Schistosoma haematobium aumenta tásica. El 50% desarrollarán metástasis a distancia, a pesar del tratamiento.
la incidencia de carcinoma escamoso vesical, así como la presencia de
infección crónica o catéter vesical permanente. El tercer grupo que merece mención aparte es el carcinoma in situ. A pe-
sar de encontrarse limitado al urotelio, por lo que es superficial, está for-
mado por células poco diferenciadas con displasia grave. Tiene una alta
Aminas aromáticas (2-naftilamina): tabaco, Transicionales (90%) tasa de recidiva y progresa hacia tumor infiltrante en el 50-75% de los ca-
industria textil, industria del caucho, colorantes Mejor pronóstico sos. Este mal pronóstico le confiere un carácter completamente distinto
Fenacetinas crónicas del carcinoma in situ de otras regiones, en las que se considera el estadio
Sacarina, cicla mato inicial de la enfermedad tumoral.
Ciclofosfamida (acroleína)
Tabaco: ortofenoles, triptófano El carcinoma in situ puede estar asociado a focos de carcinoma superficial
Schistosoma haematobium Escamosos (8%) (26%) o infiltrante (60%) o bien encontrarse de forma aislada, siendo ge-
Litiasis, infecciones, catéteres neralmente multifocal tanto en vejiga como en otros puntos del urotelio.

Cistitis glandular Adenocarcinomas (2%)


Ten en cuenta que el carcinoma urotelial que infiltra músculo lleva un pro-
Extrofia vesical
tocolo de manejo totalmente diferente al músculo invasor, es por eso que
Tabla 1. Factores etiológicos de los carcinomas del tracto urinario incluso en la literatura se manejan como dos enfermedades diferentes.

ERRNVPHGLFRVRUJ
Urología 1 06
de m icrohematuria asintomáti ca, descubierta d urante estudios de criba-
T: define el tumor do, sólo se relaciona con enfermedad significativa en menos del 2% de

Tis: carcinoma in situ (plano) los casos. Pueden encontrarse síntomas irritativos (escozor, polaquiuria,
te nesmo) en el 25-30%, solos o acompañando a la hematuria. La presen-
Ta: carcinoma papilar no infiltrante
cia de un síndrome cistítico no justificado por infección o litiasis debe
Tl : tumor que invade tejido conjuntivo subepitelial
hacer sospechar la presencia de un carcinoma vesical, especialmente por
T2: tumor que invade músculo
su asociación con el ca rcinoma in situ. Con menor frecuenc ia, el pacien-
- T2a: tumor que invade la mitad interna
te consu lta por dolor en fla nco por obst rucción ureteral, pélvico o por
- T2b: tumor que invade la mitad externa
edema en miembros inferiores (extensión linfática). La exploración física
T3: tumor que invade tejido perivesical: suele ser irrelevante, salvo en la enfermedad avanzada.
- T3a: microscópicamente
- T3b: macroscópicamente (masa extravesical) Las cito log ías urinarias son una pru eba sencilla y fiable que debe ser rea li-
T4a: tumor que invade próstata, útero o vagina za da en todos los casos de hematuria asintomática o sospecha de tumor
T4b: tumor que invade pared pélvica o pared abdominal vesical. Su sensibilidad depende del grado de diferenciación del tumor
N: define la afectación ganglionar vesica l, alcanzando el 75- 100% en t umores de alto grado y carcinoma

Nx: metástasis ganglionar regional desconocida in situ, siendo en este últi mo un método diagnóstico más rentable que
la ecog rafía, la TC, la urog rafía o la biopsia múltiple. Son especialmente
NO: ausencia de metástasis ganglionar reg ional
úti les en el seg uimiento de pacientes sometidos a resección transuretral
Nl: metástasis a un solo gangl io entre 2 y 5 cm
en combi nación con la cistoscopia.
N2: metástasis en un ganglio mayor de 5 cm o múltiples no mayo res
de 5 cm
N3: metástasis mayores de 5 cm Entre las pruebas radiológicas destacan la ecografía (con una sensibilidad
del 80%, pero poco útil para el d iag nóstico de las neoplasias del tracto
M: define las metástasis
urinario su perior [Figura 2]) y la urografía intravenosa (capaz de detectar
M0: no metástasis la presencia de tumo r en el 60% de los casos). Actualmente desplazada
M 1: metástasis a distancia por la urotomog rafia.
Tabla 2. Estad ificación del ca rci noma vesical

1. Mucosa - - - - - Ta, Tis 4. Grasa Figura 2. Ecografía. Tumor vesical con zonas calcificadas
- - -- - T3
2. Submucosa T1 S. Órganosvecinos - T4
3. Muscular - - - ~ T2 En la urografía int ravenosa, además de la presencia de defectos de re-
Figura 1. Esquema de la est ad ificació n del t um or vesica l p leción, se puede sugerir el diagnóstico de tumor vesical, rigidez y fa lta
de distensibilidad vesica l, la obstrucción de un uréter o el desplaza-
Se puede encontrar en el ámbito vesical distintas lesiones benigna s que miento de la vej iga, entre otros. Actualmente la TC helicoidal con con-
no se asocian con el desarrol lo de cáncer: los nidos de Von Brunn, la ci s- traste intravenoso en fase de eliminación aporta imágenes más preci-
titis quística y glandular originados en procesos inflamatorios o irritativos sas. En caso de dudas sobre el tracto urinario superior, se recurrirá a la
crónicos, y que pro bablemente sean distintas manifestacio nes de un mis- piel og rafía retrógrada en el momento de rea lizar la cistoscopia. Esta es
mo proceso, aunque pueden pl antear el diagnóstico diferencial; (oca sio- fu ndamental pa ra la evaluación del tumor vesical; puede rea lizarse bajo
na lmente se han descrito aden ocarcino mas vesica les asociad os a la cisti- anestes ia loca l cuando existan d udas con las pruebas rea lizadas previa-
tis glandular). Otras lesiones benignas serían el adenoma nefrogénico, el mente, pero si el d iagnóstico de presunción es firme, y dado que en
pólipo simple, el papi loma invertido o el papiloma vell oso. todo t umor debe real izarse resección t ransu retral para evaluar el grado
de infiltración, se puede esperar a t ener al paciente en quirófano bajo
Diagnóstico anestesia general o raquídea para practicarla. En un 10% de los casos de
cistoscopia con cito logías positivas no se encuentra tumo r en la vejiga,
La hematuria macroscópica o m icroscópica monosintomática es el ha- lo que puede ser debido a la presencia de carcinoma in situ, tumor en
llazg o más frecuente, presente en el 75% de los pacientes. La presencia vías urinarias altas, a un carcinoma ducta l de próstata o a un falso posi-

ERRNVPHGLFRVRUJ
Manual CTO de Medicina y Cirugía, 2.ª edición

tivo de la prueba (generalmente por inflamación de la pared vesical o dir la nefropatía por abuso de analgésicos (fenacetinas) y la nefropatía de
por tratamiento concomitante con radioterapia o quimioterapia endo- los Balcanes.
vesical).
En el 70-80% de los pacientes aparece hematuria macroscópica, siendo
La RM consigue mejores imágenes de la cúpula vesical por sus cortes el dolor cólico por obstrucción ureteral la segunda queja en frecuencia
sagitales, pero no aporta mayor información que la TC. de aparición.

Tratamiento

Todo el proceso diagnóstico va encaminado a establecer si el tumor vesi- La hematuria también es la manifestación más fre-
cal es superficial o infiltrante, ya que el tratamiento varía radicalmente en cuente del tumor maligno de riñón.
función de este hecho.

Los tumores superficiales son manejados mediante resección transure-


tral. Dada la alta frecuencia de recidivas, la mayoría se tratan posterior- Diagnóstico
mente con instilaciones endovesicales (quimioterapia o inmunoterapia
local) que disminuyan la aparición de nuevos tumores. Entre los quimio- La sistemática diagnóstica es básicamente la misma que para el tumor
terápicos empleados se encuentran la mitomicina, la tiotepa, la adriami- vesical. Es decir, ecografía (detectará hidronefrosis del lado del tumor),
cina o Ja epirrubicina. Con todos ellos se consigue reducir las recidivas UIV (se verá un defecto de repleción no compatible con un cálculo o una
alrededor de un 20%. anulación funcional de ese sistema excretor), y TC para la estadificación
(Figura 3).
La inmunoterapia endovesical con BCG (bacilo Calmette-Guerin) es, sin
duda, la más eficaz, disminuyendo el porcentaje de recidivas en un 40%,
siendo también tratamiento de elección en el carcinoma in situ, con una
eficacia del 70%. Sin embargo, no se utiliza en los tumores vesicales como
primera línea, reservándose para los tumores de riesgo o con múltiples
recidivas, ya que también se acompaña de mayor número de efectos se-
cundarios y de complicaciones, entre los que se encuentran cistitis febril,
síndrome pseudogripal, y las más graves de sepsis, prostatitis granuloma-
tosa, neumonitis e incluso muerte. Estos cuadros precisan tratamiento
tuberculostático completo al menos de tres a seis meses.

En la enfermedad invasora o infiltrante, tras la resección transuretral para


evaluar la afectación parietal, el tratamiento de elección es la cistecto-
mía radical. La quimioterapia sistémica, adyuvante o neoadyuvante, no
parece mejorar los resultados de la cirugía aislada. Igualmente se puede
decir de la radioterapia, que únicamente ha demostrado incrementar el
tiempo libre de recidiva local, sin aumentar la supervivencia.

La quimioterapia combinada con M-VAC (metotrexato, vinblastina,


adriamicina, cisplatino) se reserva para pacientes en los que existe
afectación ganglionar o metastásica, con respuestas completas en el
15-30% y parciales en el 30-40%. Tras la cistectomía, los uréteres son
derivados generalmente a segmentos intestinales o a piel, pudiendo
realizarse estomas no continentes o reservorios continentes directa-
mente al remanente uretral.

La adriamicina (doxorrubicina) es un quimioterápico


que puede producir cardiotoxicidad.

6.2. Tumores del tracto urinario superior

Entre el 2-5% de los tumores uroteliales se encuentran localizados entre (A) Urografía intravenosa con defecto de repleción en pelvis renal derecha
los cálices y los orificios ureterales. En su etiología están implicados los (B) TC en fase excretora con tumoración piélica derecha
mismos factores que para el carcinoma vesical, a los que habría que aña- Figura 3. Ecografía. Tumor vesical con zonas calcificadas

06 · Carcinomas del tracto urinario


ERRNVPHGLFRVRUJ
Urología 1 06
En estos tu mores, la citología urina ria aumenta su eficacia si se obtiene de
fo rma select iva, cateterizando el uréter del lado afectado. Ideas clave
" El carcinoma vesical más frecuente es el urotelial, siendo el ta-
Otros elementos de diagnóstico son las biopsias por cepi llado y la ure-
baco el principal factor de riesgo.
teropieloscopia, elemento que, además de diagnóstico, puede tener un
fi n terapéutico en t umores de pequeño tamaño y aspect o superfi cia l " El carcinoma escamoso se relaciona con la esquistosom iasis
(Figura 4). (S. haematobium).

" El adenocarcinoma vesical se relaciona con el antecedente de


extrofia vesical.
Fumador+ Hematuria

" El carcinoma papilar superficial y el carcinoma in situ (CIS) son


Sospecha de tumor uroteha l muy recurrentes.

" Clínica más frecuente del carcinoma urotelial: hematuria, más


t
Citologías+ Pruebas de imagen
<ECOrvp
típico con coágulos.
UIV

" Cuando se trata de un carcinoma in situ: síntomas irritativos


(polaquiuria, disuria, tenesmo, etcétera).
No concluyente Concluyente
para tumor vesical para tumor vesical
" El mejor método para la estadificación local es la resección
t t transuretral.
Cistoscopia EB - - - - - - - - - - - RTUV
e+ + + t " Prueba más sensible para el diagnóstico de CIS: citología urinaria.
Biopsia vesical EB - - - - - -Tis ;ie T2 TA, Tl

e+ + + + " Conducta ante un CIS: tratar con bacilo Calmette-Guerin (BCG)


y revisiones (cistoscopia y citologías) .
Sospechar tumor BCG Cistectomía QuimioBCG (G3)
Tracto urinario superior
· Ureterorrenoscopia + -/ + " Actitud ante un tumor superficial: resección tran suretral. Pos-
· Pielografía retrógrada Recidiva ns/cistectomfa Recidiva teriormente, revisiones (cistoscopia y citolog ías).
· Citologías selectivas
· Cepillado ureteral
" Actuación ante un tumor infiltrante (afectación capa muscu-
lar): cistectomía.
Figura 4. Algoritmo diagnóstico-terapéutico en tumor de vejiga

Casos clínicos
¿Cuál es el examen definitivo para realizar el diagnóstico de este
Masculino de 64 años de edad, tabaquismo suspendido hace 3 paciente?
meses, acude a consulta de manera subsecuente por continuar
con polaquiuria y disuria, además menciona que presentó he- 1) USG.
maturia macroscópica con presencia de coágulos la cual se auto 2) TAC.
limitó ¿cuál podría ser la causa de la patología de este paciente? 3) RMN .
4) Resección endoscópica con toma de biopsia.
1) Urolitiasis.
2) Cistitis por Klebsiella.
3) Carcinoma de células transicionales. Acorde a las estadísticas ¿cuál es el carcinoma mas frecuente del
4) Cistitis gonocócica. tracto urinario?

1) Tumor de células escamosas.


¿Cuál es el examen inicial a realizar en este paciente? 2) Carcinoma urotelial.
3) Adenocarcinoma.
1) USG. 4) Carcinoma de células pequeña s.
2) TAC.
3) RMN.
4) Cistoscopía.

ERRNVPHGLFRVRUJ
Manual CTO de Medicina y Cirugía, 2.ª edición

1
3) A urine PCR test should be performed.
Case Study 4) Urologic echography is recommended.

A 56-year-old male presents to his physician with complaints


of self-limited hematuria with no accompanying symptoms. He A patient who underwent a transurethral resection of a urinary
says he had a similar episode six weeks ago. He has been smo- bladder lesion has been subsequently prescribed BCG immu-
king more than one pack a day for 35 years. He has hypertension notherapy. Which of the following is the incorrect answer?
and is currently seeing a neurologist because he has severe hea-
daches. Regarding this patient's diagnostic workup, which of the 1) BCG immunotherapy produces only local symptoms that are
following clauses is correct? very well tolerated.
2) The mechanism of action of BCG consists of an immune-boos-
1) lf he has a urothelial tumor, cytology will be always positive. ting effect.
2) A negative echography rules out a urinary tract tumor. 3) BCG therapy should not be used in immune-deficient patients
3) A negative cystoscopy rules out the presence of a papillary tu- such as patients with AIDS.
mor in the urinary bladder. 4) BCG therapy should be indicated in patients with intermediate
4) Cytology is only indicated if we have more than one negative risk tumors and elevated risk of recurrence or progression.
cystoscopy, in order to rule out in situ carcinoma.

A 65-year-old male undergoes a transurethral resection of a uri-


A 58-year-old female complains of irritative micturition syndro- nary bladder tumor. Pathology report describes the lesion as a
me for the last few months. Several urinary cultures have been T2G3. Which of the following is the correct management of this
performed with negative results. She usually smokes, has type patient?
11 diabetes and hypertension. Urinalysis shows microhematuria.
Which ofthe following options is incorrect? 1) BCG therapy should be initiated.
2) Close follow-up.
1) lf the cytology is positive, stopping smoking would help dimi- 3) Radical cystoprostatectomy.
nish the recurrences. 4) A complete staging study should be performed.
2) A transurethral resection may be necessary in order to stage the
disease.

06 · Carcinomas del tracto urinario


ERRNVPHGLFRVRUJ
~U
~rº~'º-Q~ía___ __ •

Tumores testiculares

Este tema es sencillo y rentable siempre que se seleccione lo realmente importante: la clínica
y el diagnóstico.
En cuanto al tratamiento, dado que no existe un protocolo universalmente aceptado, es mejor
conocer ideas generales.

7 .1. Etiología y epidemiología vez alcanzada la pubertad, un teste criptorquídico no descendido pro-
bablemente deba extirparse, dado que estos testículos pierden la capa-
cidad de espermatogénesis y conservan su potencial malignizante. No
Los tumores testiculares suponen el 1-2% de las neoplasias en varones, obstante, hay autores que prefi eren no extirparlos, siempre y cuando
y son las neoplasias sólidas más frecuentes entre los 20 y 35 años. Con se puedan descender a la bolsa escrotal, ya que aún pueden mantener
un aumento en la última década en países desarrollados y una máxima su función endocrina (secretora de testosterona).
incidencia en las personas de raza blanca. Con una tasa de curación del
90%, gracias a su diagnóstico oportuno y su alta sensibilidad a la qui- Otros factores que se pueden encontrar relacionados son las hernias in-
mioterapia y radioterapia. guinales infantiles (no demostrado) y la orquitis urliana secundaria al pa-
ramixovirus causante de la parotiditis (siempre que haya producido atro-
Son tumores procedentes de células germinales en el 95%, siendo el fia) y, entre los factores tóxicos, la exposición a radiaciones, fuentes de
seminoma el más frecuente. El 5% restante se reparte entre tumores calor, productos para teñido del cuero y estrógenos intrauterino durante
del estroma gonadal (1-2%), linfomas (1 %), gonadoblastomas (células el primer trimestre del embarazo.
germinales y del estroma) metástasis y otros.
Aunque entre un 8 y 25% de los pacientes presentan historia de trauma-
Dentro de los factores de riesgos se encuentra; el síndrome de disgene- tismo testicular, todos los autores parecen estar de acuerdo en que este
sia testicular (criptorquidia, hipospadias, espermatogénesis disminuida supone más bien el motivo por el que se descubre una masa escrotal, y
o infertilidad), historia familiar de primer grado de neoplasias testicula- no su origen.
res, síndrome de Klinefelter, presencia de tumor testicular contra lateral,
neoplasia intraepitelial testicular, etc. Dentro de los factores más impor-
tantes se encuentra Ia criptorquidia (kriptos-ocuIto, orquis-testícuioi, 7 .2. Anatomía patológica
con un aumento en la incidencia de entre 2 a 8 veces más con el resto
de la población y siendo la situación intraabdominal del teste el factor
de mayor riesgo. El teste contralateral, aunque de localización escrotal, Los t umores testiculares se suelen originar como una masa testicular in-
también tiene un aumento en la incidencia, pudiendo presentarse has- traparenquimatosa. A partir de ahí, pueden quedarse localizados o me-
ta en un 20% de los casos el tumor en el teste intraescrotal. Es por esto tastatizar. Ocasionalmente surgen de forma primaria en retroperitoneo,
que la anamnesis del recién nacido es obligada, y se debe hacer una med iastino y muy rara vez en la glándula pineal (tumores germinales
evaluación minuciosa de la presencia de los testes en la bolsa escrotal. extragonadales). La localización más frecuente de metástasis son los
ganglios retroperitoneales (casi siempre es el primer escalón en la dise-
A la fecha la exploración clínica continúa siendo el método de diagnós- minación); posteriormente puede aparecer la afección mediastínica, de
tico más importante de la criptorquidia, sin embargo en exploraciones vísceras abdominales y de pulmón.
difíciles se puede realizar ecografía para apoyar el diagnóstico. De notar
la ausencia de una, iniciar el protocolo para lograr el descenso entre el Hay que tener en cuenta que el testículo izquierdo drena a los gan-
primer y segundo año de edad. El descenso del teste no disminuye el glios paraaórticos y preaórticos a nivel de L2. El derecho drena a gan-
riesgo de cáncer como tal, pero si facilita su pronto diagnóstico. Una glios interaortocavos, precavos y preaórticos, y también a nivel del

ERRNVPHGLFRVRUJ
Manual CTO de Medicina y Cirugía, 3.ª edición

hilio renal. La diseminación hematógena es menos frecuente, salvo Teratocarcinoma. Tumor mixto con áreas de teratoma y de carcino-
en el coriocarcinoma, vía vasos espermáticos, siendo los puntos más ma embrionario. Un 64% tiene también áreas de seminoma.
habituales de metástasis: pulmón, hígado, hueso y SNC, por orden de Gonadoblastoma. Contiene grandes células similares al seminoma
frecuencia. y otras menores, como células de Sertoli inmaduras o de la granulo-
sa. Se asocia preferentemente a las gónadas disgenéticas y estados
Los tumores testiculares se dividen en dos grupos: tumores que no de- intersexuales.
rivan de las células germinales (S%) y tumores que derivan de las células Tumores de células de Sertoli. Forman áreas tubulares similares a
germinales (9S%); a su vez, estos últimos se dividen en seminomatosos y los tú bulos del testículo prepuberal normal. De comportamiento be-
no seminomatosos (Tabla 1). Los no seminomatosos presentan un creci- nigno, es excepcional que metastaticen.
miento más rápido y tendencia a dar metástasis. Por su parte los tumores
de células no germinales o del estroma tienen su origen en las células de
Leydig y Sertoli y generalmente son benignos.
El tumor de células de Sertoli se ha asociado al sín-
drome de Peutz-Jeghers
Tumores de células germinales Tumores del estroma
Tumores de un tipo histológico Tumores células Leydig
Semi noma: Tumores células Sertoli Tumores de células de Leydig. Pueden verse cristaloides en su
Típico Tumores estructuras citoplasma. Generalmente son benignos. Pueden ser hormonal-
gonadales primitivas mente activos, produciendo pubertad precoz o feminización.
- Anaplásico
linfoma. Tanto como localización secundaria o como origen pri-
- Espermatocítico
mario (menos frecuente), una masa testicular en un varón mayor
Carcinoma embrionario de SO años sugiere, en primer lugar, el diagnóstico de linfoma.
Poi iem brioma Tras la orquiectomía o la biopsia testicular para el diagnóstico
Tumor del saco vitelino (seno endodérmico) de certeza, el tratamiento no varía respecto a los linfomas habi-
Coriocarcinoma tuales.
Teratoma:
- Maduro
- Inmaduro 7 .3. Clínica
- Con transformación maligna
Tumores de más de un tipo histológico
La presencia de una masa testicular sólida sin dolor es patognómoni-
Teratocarcinoma
co de tumor de testículo. Puede presentarse dolor en escroto como
Otros
primer síntoma (20%), dolor local (27%) y dolor de espa lda y flanco
Tumores germinales + estromales
(11 %). En algunos casos puede haber ginecomastia o bien con un
Gonadoblastoma cuadro similar a una orquiepididimitis, presentándose como síndro-
Tabla 1. Tumores de testículo. Clasificación histológica me de escroto agudo hasta en 10% de los pacientes.

Semi noma. Puede aumentar el tamaño testicular hasta 1O veces


sin distorsionar su morfología. Ocasionalmente se presenta extra-
Existen algunas enfermedades, como la sarcoido-
testicular en mediastino (la más frecuente), retroperitoneo, o re- sis, que aumentan el tamaño testicular sin existir
gión pineal de forma primaria. Se han descrito tres tipos histológi- un tumor.
cos: seminoma típico, anaplásico y espermatocítico (más frecuente
en individuos mayores de SO años). El seminoma puro no es pro-
ductor de marcadores tumorales, pero hasta en un 1S% de los ca-
sos pueden aparecer células del sincitiotrofoblasto, produciendo 7 .4. Diagnóstico
elevaciones de la P-HCG.
Carcinoma embrionario. Tiende a metastatizar de forma temprana.
Forma masas más irregulares y heterogéneas que otros tumores. La exploración física de una masa indolora, sin signos inflamatorios y de
Tumor del seno endodérmico. Este tumor fue descrito inicialmen- largo periodo de evolución, debe hacer sospechar un tumor testicular
te como una forma rara de tumor infantil. Se encuentran elementos (véase la Tabla 2).
de saco vitelino en el 38% de los tumores testiculares del adulto.
Coriocarcinoma. En el estudio histológico tiene que contener ele- Un elemento importante, tanto para su diagnóstico como para el segui-
mentos de sincitiotrofoblasto y citotrofoblasto para ser considerado miento, son los marcadores tumorales: a-fetoproteína (AFP) y fracción p
como tal. de la gonadotropina coriónica humana (P-HCG) (Tabla 3).
En el momento del diagnóstico, generalmente existen metástasis a
distancia (suelen ser vía hematógena) y un tumor primario testicular La AFP es sintetizada por células del saco vitelino y, por tanto, está pre-
pequeño. Raramente es puro. sente en tumores de saco vitelino o seno endodermal, y en los carcino-
Teratoma. Por definición, se encuentra formado al menos por dos mas embrionarios. El seminoma nunca produce AFP. En cualquier caso,
capas distintas de células germinales (endodermo, mesodermo o se debe tener en cuenta que la AFP es un marcador inespecífico, y se
ectodermo). podría encontrar elevado en enfermedades hepáticas benignas o ma-

07 · Tumores testiculares
ERRNVPHGLFRVRUJ
Urología 1 01
lignas, algunas neoplasias pancreáticas y de la vía biliar o en la ataxia- La vida media de la a-fetoproteína es de siete días, frente a tres días de la
telangiectasia. ~-HCG. Este dato es importante a la hora de valorar la posible presencia
de enfermedad residual, ya que la elevación persistente de uno de estos
marcadores después del tratamiento supone la existencia de tumor no
eliminado.
La AFP también se eleva en líquido amniótico en al-
teraciones del cierre del tubo neural. La ecografía testicular es un método sencillo y fiable para la diferencia-
ción entre masas sólidas y quísticas, y su localización exacta intratesticu-
lar o dependiente de los anejos. Cuando, a pesar de los marcadores, los
La fracción ~dela HCG es producida por las células del sincitiotrofoblasto datos ecográficos son sugerentes de tumor, está indicada la exploración
presentes en el coriocarcinoma y, también de forma ocasional y de forma quirúrgica a través de una incisión inguinal, para evitar la posibilidad teó-
aislada, en algunos seminomas. rica de implantes tumorales en la piel escrotal y tener mejor control de
pedículo vasculolinfático a nivel del cordón espermático.
En conjunto, el 70% de los tumores testiculares producen algún marca-
dor, luego existe hasta un 30% de tumores con marcadores negativos al Si la exploración confirma la presencia de una masa, el testículo debe ser
diagnóstico. extirpado de forma urgente (orquiectomía radical).

La evaluación de la extensión tumoral se completará mediante TC tora-


pTis lntratubular coabdominal. Así se constatará si la enfermedad está limitada al testículo
pTl Testículo y epidídimo sin invasión vascular/linfática (estadio 1), o existe afectación de ganglios infradiafragmáticos (estadio 11),
pT2 Testículo y epidídimo con invasión vascular/linfática o túnica o bien si hay incluso extensión supradiafragmática o a órganos sólidos
vaginal (estadio 111). El sistema de estadificación utiliza múltiples variaciones, pero
pT3 Afectación de cordón espermático quizá la clasificación más aceptada sea la expuesta anteriormente en la
pT4 Escroto Tabla 2.

Nl Ganglios menores de 2 cm
N2 Ganglios entre 2-5 cm
N3 Ganglios mayores de 5 cm
7 .5. Diagnóstico diferencial
Mla Metástasis en ganglios no regionales o pulmón
Ml b Metástasis viscerales no pulmonares Aunque frecuentemente la existencia de un tumor testicular no plantea
Sx Marcadores tumorales séricos no disponibles dudas diagnósticas, existen una serie de patologías testiculares que, jun-
SO Niveles de marcadores normales to con los tumores, pueden presentarse en algún momento dentro del
51 LDH < 1,5 x n; y ¡3-HCG < 5.000; y AFP < 1.000 cuadro genérico que se denomina "escroto agudo''. caracterizado por el
52 LDH entre 1,5 x n y 1Ox n o ¡3-HCG entre 5.000 y 50.000 aumento doloroso de volumen del contenido escrotal, acompañado o
o AFP entre 1.000 y 10.000 no de signos inflamatorios (Figura 1 y Tabla 18).
53 LDH > 1Ox n o ¡3-HCG > 50.000 o AFP > 10.000
n = límite superior normal de LDH
Agrupación por estadios TMR testículo (abreviado)*
Estadio O pTis NO MO
Estadio 1 pTl -4 NO MO
Estadio IIA Cualquier pT Nl MO
Estadio IIB Cualquier pT N2 MO
Estadio IIC Cualquier pT N3 MO
Estadio 111 Cualquier pT Cualquier N Ml
* No tiene en cuenta marcadores tumorales
Tabla 2. Estadificación clínica y patológica

Tumor con una palabra aumenta ¡3-HCG


- Coriocarcinoma
- Seminoma (en la mujer, el equivalente es el disgerminoma)

Tumor con dos palabras aumenta a-fetoproteína


- Carcinoma embrionario
- Tumor del seno endodérmico

Tabla 3. Marcadores tumorales en tumores germinales (válidos para


ovario y testículo) Figura 1. Pieza macroscópica de tumor testicular

ERRNVPHGLFRVRUJ
Manual CTO de Medicina y Cirugía, 3.ª edición

Orquitis Polaridad conservada Estadio lla-llb. En este caso, el tumor ya está extendido a ganglios
Prehn MEJORA el dolor retroperitoneales, y requiere, por tanto, tratamiento agresivo. Se
Torsión del cordón Testículo horizontalizado dispone de dos opciones: radioterap ia sobre las cadenas afectadas
espermático Prehn EMPEORA el dolor (teniendo en cuenta el teste afectado, se irradiará a unas cade nas
ganglionares u otras), o quimioterapia BEP (cisplatino, etopósido y
Tumor testicular Masa palpable indolora
bleomicina). Ambos obtienen resultados muy sim ilares.
Tabla 18. Diagnóstico d iferencial de los tumores testiculares Estadio llc-111 (estadios avanzados). El tumor tiene metástasis gan-
glionares retroperitoneales superiores a 5 cm, o afectación supra-
Orquiepididimitis: suelen presentarse con dolor intenso, enroje- diafragmática o de vísceras sólidas. La radioterapia deja de ser una
cimiento cutáneo, fiebre, y a veces sintomatología miccional. En la opción terapéutica. La quimioterapia (BEP) es la única posibilidad,
exploración, la elevación del testículo (signo de Prehn) disminuye el presentando una tasa de curación de alrededor del 80%.
dolor (signo de Prehn positivo). En su etiología se deben conside rar
gérmenes de transmisión sexual en pacientes adultos menores de Tumores no seminomatosos
35 años, y uropatógenos (E. coli) si superan esta edad.
Torsión del cordón espermático: suele aparecer en la infancia o Estadio l. Existen tres posibilidades terapéuticas tras la orquiectomía:
la adolescencia, con dolor de aparición brusca y signos cutáneos Observación y seguimiento periódico. Tasas de recaídas de un
inflamatorios crecientes a medida que progresa el cuadro. El teste 20%.
se encuentra horizontalizado, y ocasionalmente puede palparse la Linfadenectomía retroperitoneal de estadificación, pues así se
espiral del cordón torsionado. En este caso, la elevación del testículo tiene certeza del estadio y se reduce a un 10% las recidivas (en
incrementa la sensación dolorosa. Europa no está extendida esta práctica).
Hidrocele y espermatocele: son dos cuadros que raramente se Quimioterapia profi láctica (cisplatino), y así se reducen al 5% las
presentan de forma brusca y con dolor agudo, fácilmente diferencia- recidivas.
bles por la exploración y su transiluminación positiva y, ante la duda,
mediante ecografía. En aquellos casos en que exista invasión vascular en la pieza de or-
quiectomía, la tasa de recidivas asciende al 50%. Parece lógico, en es-
tos casos, inclinarse de entrada por una de las dos últimas opciones.
7.6. Tratamiento La tasa de curación alcanza el 98%.
Estadio lla-llb. Históricamente se realizaba linfadenectomía retro-
peritoneal comp leta como único tratamiento, pero ante tasas de
Hasta la fecha no existe un protocolo único de t ratamiento, pudiendo va- recidiva no desdeñables, actua lmente se indica quimioterapia de
riar incluso de un centro a otro. Siempre se realizará orquiectomía radical inicio (BEP). La tasa de supervivencia supera el 95%.
vía inguinal. Posteriormente, el patólogo informará de la estirpe histopa- Estadio llc-111. Antes de la aparición de la actual quimioterapia, la su-
tológica. A continuación, el estudio de extensión mediante TC toracoab- pervivencia era del 5-10%. Actualmente, el esq uema de tratamiento
dominopélvica y nuevos marcadores postorquiectomía. En función de la mayoritariamente aceptado es quimioterapia primaria.
histología y del estadio, se asignará un tratamiento a cada paciente.
El tratamiento de la neoplasia testicular se resume en la Figura 2.

La orquiectomía en el cáncer de testículo es vía ingui-


nal y de carácter urgente. En el cáncer de próstata, vía Tumor testicular
escrotal y con cirugla programada

Orquiectomía radical
Seminoma

Se caracteriza por su gran radiosensibilidad, de ahí que la radioterapia


haya sido la base del tratamiento de estos tumores. Actua lmente, la qu i-
t t t t
lla,llb la lla,llb
mioterapia obtiene resultados similares.
Estadio l. El tumor teóricamente está limitado al testículo. No obstan-
· Observación
· Si FR:
t · Observación
· Linfadenectomía
+++
invasión
t
Quimioterapia
Radioterapia
te, se sabe que hasta un 20% de pacientes presentan micrometástasis > 4 cm rete testis Quimioterapia
(EEUU) vascular
· Quimioterapia
a ganglios retroperitoneales, que en el momento del diagnóstico no t (UE)
o
linfática
se detectan. La presencia de micrometástasis se ha relacionado con Radioterapia
dos factores de riesgo: tumor de más de 4 cm y la invasión tumora l de Quimioterapia

la rete testis. En estos pacientes se aconseja administrar radioterapia o ~~ ~ ~ .'


quimioterapia (cisplatino). Con esto, frente a los pacientes que opta- ~;~ ~~ ~ /
ron únicamente por observación, la recidiva desciende del 20 al 5%.

Quimioterapia

El cisplatino produce vómitos con mucha frecuencia.


Otro efecto secundario es su nefrotoxicidad. Figura 2. Algoritmo de tratamiento de la neoplasia testicular

07 · Tumores testiculares
ERRNVPHGLFRVRUJ
Urología 1 01
Masas residuales vertirse en teratomas malignos o producir procesos compresivos con su
crecimiento) (Figura 3).
Se define como masa residual la existencia de conglomerados adenopá-
ticos tras tratamiento quimioterápico o radioterápico. Cuando el tumor
primario es un semi noma y existen masas residuales, la actuación a seguir
será: si la masa residual es inferior a 3 cm, tiene muy pocas probabili-
1%Hiii+ii
dades de contener tumor residual y no requiere más que observación. t t
Pero cuando es superior a 3 cm, se debe realizar una PET (tomografía por
emisión de positrones), si existiera esta posibilidad a nivel técnico, pues
detecta con una alta sensibilidad y especificidad la presencia de tumor t
<3cm
t
>3cm
residual. Si no se dispone de una PET o si esta es positiva, se realizará
cirugía de la masa.

Observación
Cuando el tumor primario es un tumor no seminomatoso, se debe reali-
zar exéresis de dicha masa siempre, con independencia del tamaño.
Exéresis de la masa residual

La histología de estas masas residuales, una vez extirpadas, puede ser:


50% necrosis 35% teratoma 15% tumor viable
tejido necrótico hasta en un 50% de las ocasiones, tumor viable en un
15% y teratoma en un 35% (estos, dejados a libre evolución, pueden con- Figura 3. Algoritmo de tratamiento de las masas residuales

" La clínica más frecuente es una masa escrotal indolora.


Ideas clave
" El seminoma nunca produce a-fetoproteína.
" El tumor testicular es la neoplasia sólida más frecuente en el va-
rón joven. " Como tratamiento, la orquiectomía por vía inguinal se realiza en
todos los casos.
" La tasa de curación es mayor del 90%.
" El seminoma es radiosensible. Por ello puede utilizarse radiote-
" El tumor testicular más frecuente es el seminoma. No obstante, rapia como tratamiento en los primeros estadios. Si se trata de
esto es muy variable según la edad del paciente. un estadio avanzado, se emplearía quimioterapia.

" Una masa testicular por encima de los 50 años debe hacer pen- " Los tumores no seminomatosos se tratan con orquiectomía +
sar en un linfoma. quimioterapia. No obstante, si es un tumor limitado al testículo,
puede plantearse la vigilancia tras la orquiectomía.
" El tumor típico de las disgenesias gonadales es el gonadoblastoma.

1) Vigilancia.
Casos clínicos 2) Radioterapia.
3) Quimioterapia asociada a Radioterapia.
Masculino de 21 años de edad, con antecedente de importancia 4) Quimioterapia.
criptorquidia, acude por detectarse crecimiento testicular dere-
cho, no doloroso a la exploración. ¿Cúal es la probable etiología
del padecimiento de su paciente? Un paciente de 35 años, con insuficiencia renal crónica, secun-
daria a pielonefritis recibe un primer transplante renal de vivo
1) Orquiepididimitis. relacionado. ¿Cuál de las siguientes terapias inmunosupresoras
2) Torsión del cordón espermático. es la más indicada para este paciente?
3) Hidrocele.
4) Seminoma. 1) Ciclosporina + micofenolato + prednisona.
2) AntiCD25 + esa.
3) Csa.
Posterior a orquitectomía radical derecha se confirma semi noma 4) Tac+ micofenolato + prednisona.
y se estatifica en una etapa clínica I sin invasión a rete testis, tu-
mor de 3 cm. ¿Cuál es el tratamiento indicado en este caso?

ERRNVPHGLFRVRUJ
Manual CTO de Medicina y Cirugía, 3.ª edición

Which of the following options is the most likely diagnosis of


Case Study a patient presenting with a scrotal mass that feels like a bag of
worms?
A 35-year-old male presents to your outpatient clinic with a
three-month history of painless swollen testicle. Tumor markers 1) Pure semi noma.
a.-FP y ~-HCG are elevated. Which of the following is the most 2) Varicocele.
appropriate answer about this case? 3) Hydrocele.
1) Testicular torsion.
1) The most likely diagnosis is a pure semi noma.
2) The most likely diagnosis is a yolk sac tumor.
3) Tumor markers can be used for follow-up. A 32-year-old patient reports to the physician a three-week his-
4) The elevated ~-HCG is directly related tó the presence of hema- tory of palpable lump in his testicle. Physical examination re-
togenous metastatic lesions. veals increased size of the right testicle with a painless petrous
nodule. Testicular echography reveals an intraparenchymatous
lesion that measures 2 cm and has no echogenicity. Which of the
A 32-year-old patient presents to his physician saying that his following tests or maneuvers would not be indicated in this pa-
testicles have increased in size for the last two months. Physical tient?
examination reveals a palpable lump in his right testicle. Ultra-
sound shows an intraparenchymatous lesion with calcifications. 1) Thoraco-abdomino-pelvic CT sean in order to stage the di sea se.
Mark the correct answer: 2) A blood sample in order to determine ~-HCG levels.
3) Schedule a radical orchiectomy as soon as possible.
1) An inguinal orchiectomy is indicated. 4) Perform a bilateral testicular biopsy.
2) A transscrotal orchiectomy is indicated.
3) A contralateral testicle biopsy should be performed.
4) The patient should receive neoadjuvant chemotherapy.

07 · Tumores testiculares
ERRNVPHGLFRVRUJ
Urolog_ía _ _

Trasplante renal

Las contraindicaciones absolutas son las siguientes:


Lo más importante de este tema
Infección activa (Cribado para VHB, VHC, VIH, CMVYTB).
ENARM son los tipos de rechazo. Se debe
insistir en el rechazo agudo. Enfermedad maligna que no pueda ser erradicada.
Sospecha de no cumplimiento terapéutico del protocolo inmuno-
supresor.
Glomerulonefritis activa.
Expectativa de vida reducida por enfermedad de base del paciente.
8.1. Indicaciones Presencia de anticuerpos preformados frente a antígenos del do-
nante.

Las dos enfermedades que más comúnmente abocan a una insuficiencia En referencia a las contraindicaciones relativas del trasplante renal se
rena l terminal irreversible, tratable mediante un trasplante renal, son la debe decir que estas se han ido modificando a lo largo de los años, gra-
glomerulonefritis y la diabetes mellitus insulinodependiente. cias a la mejora en las terapias de inmunosupresion y al trabajo multidis-
ciplinario que han mejorado la técnica y los cuidados prequirúrgicos y
Otras causas importantes son: posquirúrgicos En muchas ocasiones, el trasplante plantea menos riesgo
Poliquistosis renal. que una hemodiálisis crónica.
Nefroesclerosis hipertensiva.
Enfermedad de Alport. Actualmente se consideran contraindicaciones relativas la edad avan-
Nefropatía lgA. zada, la oxalosis, la amiloidosis, la enfermedad iliofemoral oclusiva, las
Lupus eritematoso sistémico. anomalías del tracto urinario inferior o las alteraciones psiquiátricas gra-
Nefroesclerosis. ves. La recidiva histológica de la nefropatía original es poco frecuente
Nefritis intersticial. (< 10%), pero deben considerarse, tal es el caso de la enfermedad por
Pielonefritis. depósito de cadenas ligeras, oxalosis y anticuerpos anti-membrana ba-
Uropatía obstructiva. sal glomerular.

Los mejores receptores son individuos jóvenes cuyo fallo renal no se


deba a una enfermedad sistémica que pueda dañar el riñón trasplantado 8.2. Complicaciones
o causar la muerte por causas extrarrenales. Generalmente se suele man-
tener al receptor en tratamiento con diá lisis durante un cierto tiempo
previo al trasplante. Las complicaciones que se pueden presentar son las siguientes:
Rechazo (Tabla 1).
Recurrencia de la enfermedad en el riñón trasplantado.
Complicaciones técnicas. Complicaciones vasculares, hemorra-
La nefrectomia laparoscópica para la donación es el
gold standard y ha venido a aumentar la tasa de do- gia, hipertensión por estenosis de la arteria renal, trombosis veno-
nador vivo. sa, complicaciones del tracto urinario, (estenosis ureteral, reflujo re-
noureteral), necrosis tubular aguda, linfoceles y abscesos de pared.

Contraindicaciones
En pacientes con diabetes insulinodependiente se
El trasplante renal es un procedimeinto multidisciplinario, el cual invo- debe considerar un trasplante renal y de páncrea . en
lucra a urólogos, angiólogos, psiquiatras, anestesiológos, entre otros, los pacientes con oxalosis, cosiderar un trasplante com-
binado de hígado y riñon.
cuales deben valorár si el paciente es apto para recibir un trasplante.

ERRNVPHGLFRVRUJ
Manual CTO de Medicina y Cirugía, 3.ª edición

Complicaciones no técnicas. Infecciones bacterianas y oportunis- guardan relación con el tratamiento inmunosupresor). Puede apa-
tas en relación con la inmunosupresión, hiperglucemias, complica- recer hipertensión debida a enfermedad en los riñones origina les,
ciones gastrointestinales, hiperpa ratiroidismo y tumores (cáncer de como consecuencia de rechazo, por estenosis de la anastomosis de
piel y de labios, carcinoma in situ de cérvix, linfomas no Hodgkin; la arteria rena l o por toxicidad rena l por ciclosporina.

N;M,fo·M-- Patogenia PA Tratamiento


Hiperagudo Minutos, días Ac. preformados Trombosis microvascu lar Nefrectomía del injerto
CID Isquemia o infarto
Act. del complemento PMN en capi lares
Daño endoletelial
Acelerado Días Celular (+Ac) Vasculitis necrotizante Bolos de esteroides
Respuesta 2.ia a Ag-H LA Ac monoclonales
Agudo Semanas Celular (+Ac) Forma vascu lar: mediada por Ac Bolos de esteroides (la vascular suele ser resistente)
Infiltrado de linfocitos Forma celular: tubulointersticial Ac monoclona les
Crónico Meses, años Hu moral y celu lar [ntima arterial aumentada No hay; cont rol de HTA
Atrofia tubular
Glomerulopatía
Tabla 1. Rechazo en el trasplante renal

" La clínica característica del rechazo agudo es: fiebre, hiperten-


Ideas clave si ó n y dolor en el área del injerto.

" Las causas más frecuentes de insuficiencia renal crónica son la " El rechazo agudo produce oliguria, no poliuria.
diabetes mellitus y las glomerulonefrit is.

presenta fiebre de 38 °C, TA de 180/11 O, oliguria y disminución


Casos clínicos en la concentración urinaria de sodio. El diagnóstico más pro-
bable sería:
Un paciente de 35 años, con insuficiencia renal crónica, secun-
daria a pielonefritis crónica recibe un trasplante renal de cadá- 1) Infección respiratoria.
ver con el que compartía dos identidades en A y By una en DR. 2) Pielon efritis ag uda del injerto renal.
Recibe tratamiento inmunosupresor con ciclosporina A y cor- 3) Recid iva de su enferm edad ren al.
ticoides en dosis estándar. En el posoperatorio inmediato se 4) Rechazo agudo del injerto rena l.
observa buena diuresis, y no es necesario el tratamiento susti-
tutivo con hemodiálisis. En el 5. 0 día de evolución, el paciente

08 · Trasplante renal
ERRNVPHGLFRVRUJ
U1ología

Uropatía obstructiva

Tiempo de evolución. La aguda suele cursar con dolor (cólico nefrí-


Se debe tener una idea general tico), siendo la crónica más frecuentemente asintomática.
y aprenderse bien las Ideas Lugar de obstrucción:
Clave. Tracto urinario inferior (uretra y vejiga). Cursa con retraso
para iniciar la micción, disminución de fuerza y del tamaño del
chorro, goteo terminal, hematuria, escozor al orinar, orina turbia,
retención aguda de orina o incontinencia paradójica ("micción
9.1. Características por rebosamiento").
Tracto urinario superior (uréter y riñón). Estos pacientes pre-
sentan dolor en el flanco (riñón y uréter proximal), dolor en flan-
Detención del flujo de orina en cualquier punto entre los cálices re- co con irradiación a genitales (uréter medio) o síndrome miccio-
nales y el exterior (Tabla 1). nal (uréter terminal).
Su importancia reside en el desarrollo potencial de insuficiencia re-
nal, por lo que tiene importancia la obstrucción urinaria bilateral o la Después de resolverse una obstrucción, sobre todo si es crónica, puede
unilateral sobre riñón único funcionante. producirse una fase de poliuria.
Una obstrucción de más de un mes de duración puede dar lugar
a un daño renal funcional y estructural permanente. Esto se debe a que, a nivel tubular, cuando se ralentiza crónicamente el
flujo urinario, se genera una insensibilidad a la ADH transitoria (diabetes
insípida nefrogénica), de ahí la poliuria.

La uropatía obstructiva puede producir glomerulone-


fritis focal y segmentaria.
En pacientes menores de 50 años sospechar de una
litiasis renoureteral, en pacientes mayores de 50 años
sospechar un crecimiento prostático.

9.2. Clínica
9.3. Diagnóstico
La forma de presentación depende de los siguientes factores:
Etiología de la obstrucción. Presenta la clínica propia de la enfer- El diagnóstico de seguridad, la valoración de la evolución, y el pronós-
medad de base. tico son ecográficos, aunque en muchas ocasiones se require una to-

lntraluminal lntraparietal Extraparietal (compresión extrínseca) Extraparietal (disfunción neuromuscular)

Litiasis Estenosis congénita Uréter retrocavo Vejiga neurógena


Tumores (hipernefroma, Estenosis postinfecciosa Riñón en herradura Vejiga automática: lesión sobre L1-L3
urotelioma) Estenosis postraumática Fibrosis retroperitoneal Vejiga átona: lesión bajo L1-L3
Necrosis papilar Estenosis isquémica Tumores (próstata, vejiga, ginecológicos) Disfunción de la unión pieloureteral
Coágulos Hiperplasia prostática Reflujo vesicoureteral
Ligadura iatrógena de uréteres

Tabla 1. Clasificación de la uropatía obstructiva

ERRNVPHGLFRVRUJ
Manual CTO de Medicina y Cirugía, 3.ª edición

mografía para identificar la causa de la obstrucció n y t omar decisiones


t erapéuticas. Manejo de la uropatia obstructiva

Además, son útiles la anamnesis y la exploración física, la radiología simple t


lnfravesical
t
Supravesical
(conveniente en litiasis radioopaca), urografía intravenosa (confirma una (globo vesical) (no globo vesical)

.,.
posible anomalía funcional y útil en lit iasis radiotra nsparente), cistografía,
estudio metabólico (útil en prevención de recidivas), TC abdominal, eco- t t
grafía t ra nsrectal, biopsia prostática dirigida, pielografía retrógrada, ne-
frostog rafía, cistoscopia, flujometría, cistomanometría y citología urinaria.

Dilatación unilateral
t
Dilatación

~
bilateral
9.4. Tratamiento
j
Estudio médico Birreno Monorreno

Es necesario restablecer el fl ujo urinario. La mayor parte de las veces se Estudio (UIV, TC)
~

L
Fracaso renal
rea liza mediante manejo endoscopico (litotricia, lito lapaxia, resección
transuretral o colocación de cáteteres) o correción quirúrgica (ureteroli-
Derivactndevía
totomia, prostatectomia suprapúbica, etc). urinaria superior
(nefrostomía/doble J)
Si la obstrucción es aguda y/o bilateral, la desobstrucción es urgente, y
puede lograrse mediante un sondaje vesical, talla vesical, catéter ureteral Figura 1. Procedimiento de actuación frente a la uropatía o bstructiva
o nefrostomía, sobre todo cuando se sospeche de una sobreinfección del
padecimiento de base, lo que obligue a diferir el tratamiento definitivo.
Pacientes con litiasis renoureteral que presenten da-
En caso contrario, hay que valorar el grado de sufrimiento renal y su re- tos de infección (ego patológico, leucocitosis, fiebre),
versibilidad. En los casos en que hay dest rucción irreversible de la vía uri- solo derivar la vía urinaria y d iferir tratamiento defi-
nitivo.
naria, es necesario rea lizar una derivación urinaria defi nitiva {Figura 1).

" El dolor suele estar presente en la obstrucción aguda. Sin em-


Ideas clave // bargo, en la crónica, es frecuente su ausencia.

" La uropatía obstructiva puede producir insuficiencia renal si no " Después de resolver una uropatía obstructiva puede producirse
se resuelve a tiempo. una fase de poliuria.

years ago he felt similar symptoms and had a notable improve-


Case Study ment after initiating treatment with finasteride. In this case:

A 78-year-old patient is brought to the emergency department 1) A suprapubic cystostomy should be perform ed.
with complaints of urinary incontinence. On the one hand, he 2) Foley catheterization should be tried.
says he is no trouble urinating when he wants to and on the other 3) Anticholinergic agents are ind icated.
hcmd he says he has constant urinary leakages all day long. A few 4) Pelvic floor muscle training exercises are recommended.

09 · Uropatía obstructiva
ERRNVPHGLFRVRUJ
Urologú_ _ _ _~ .

Disfunción eréctil

sentar DE. Están implicados mecanismos vasculares, neuropáticos y


Se debe incidir en los factores
de riesgo, el tratamiento disfunción gonadal.
ENARM y sus contraindicaciones. Enfermedad cardiovascular: cardiopatía, hipertensión arterial, en-
fermedad vascular periférica y descenso del colesterol HDL se han
relacionado de manera clara con la disfunción eréctil.
Tabaquismo: factor independ iente.
10.1. Introducción Secundaria a fármacos: aquellos que causan hiperprolactinemia,
que disminuyen los niveles de testosterona, psicotropos y antihiper-
tensivos.
La disfunción eréctil (DE) se define como la incapacidad persistente o re- Secundaria a consumo de drogas: cocaína, heroína, etcétera.
currente para conseguir o mantener la suficiente rigidez del pene que Trastornos afectivos: depresión.
permita una relación sexual satisfactoria, con un impacto fisico y psicoso-
cial que repercute en su calidad de vida. Debe tener una duración míni-
ma de tres meses.
La disfunción erecti l es un factor de riesgo para desa-
rrollar enfermedad cardiovascular.
10.2. Prevalencia

En Estados Unidos, en un estudio en varones de entre 40 y 70 años, se 1 0.S. Diagnóstico


estimó que la prevalencia global era del 52%.

El diagnóstico debe basarse en los siguientes componentes:


10.3. Etiología Historia clínica y sexual: investigar los posibles factores de riesgo
implicados.
Exploración física: encaminada a descartar enfermedad vascular,
Se puede clasificar en: enfermedades neurológicas, trastornos genitales y endocrinopatías.
Orgánica. Causas vasculares (las más frecuentes 60-80%), neurológi- En varones mayores de 50 años se incluirá tacto rectal.
cas (10-20%), hormonales (5-10%) o locales. Determinaciones analíticas: glucemia basal, perfil lipídico, testos-
Psicógena. terona total y libre y prolactina. Además, es conveniente solicitar he-
Mixta. En la mayoría de los casos de etiología orgánica se añade un mograma, función renal y hepática.
componente psicológico. Pruebas especializadas: únicamente en ocasiones muy seleccio-
nadas.

10.4. Factores de riesgo


10.6. Tratamiento
Edad: factor independiente.
Diabetes: es la enfermedad endocrina más frecuente asociada a dis- Se puede estructurar en tres escalones o etapas, que serán superadas
función eréctil. Significa una probabilidad tres veces superior de pre- en función de fracaso del escalón previo.

ERRNVPHGLFRVRUJ
Manual CTO de Medicina y Cirugía, 3.ª edición

Fármacos orales Apomorfina: agonista dopaminérgico que actúa a nivel central so-
bre el mecanismo de la erección.
lnhibidores de la fosfodiesterasa S (sildenfafilo, vardenafilo, tada- Está contraindicado en sujetos que tengan desaconsejada la activi-
lafilo): son el tratamiento de elección. Induce la relajación de múscu- dad sexual.
lo liso del cuerpo cavernoso, liberando óxido nítrico (NO). El NO es el
principal neurotransmisor de la erección. Precisa de deseo sexual y Segunda línea
estimulación previa para que tenga efecto.
Las contraindicaciones absolutas de sildenafilo son: Terapia intracavernosa: alprostadil (PGEl), mediante inyección di-
Administración concomitante con nitratos o fármacos donado- recta en los cuerpos cavernosos. Otros fármacos son la papaverina y
res de óxido nítrico por el riesgo de hipotensión grave (dinitra- la fentolam ina.
to/mononitrato de isosorbida, molsidomina, nicorandil, nitrogli-
cerina, nitroprusiato sódico). Tercera línea
Pacientes en los que esté desaconsejada la actividad sexual
(angina inestable, insuficiencia cardíaca o infarto reciente, hace Cirugía de revascularización (venosa, arterial). Implante de pró-
menos de seis meses). tesis de pene._

Vasculares

Enfermedad cardiovascular (cardiopatía isquémica, hipertensión, vasculopatía periférica).


Diabetes mellitus.
Hiperlipidemia.
Tabaquismo.
Cirugía pélvica mayor (OR) o radioterapia.
Neurogenica

Causas centrales
Desordenes degenerativos (esclerosis múltiple, enfermedad de Parkinson, etc.)
Trauma o enfermedad medular.
Infarto (EVC).
Tumores del sistema nervioso central.
Causas periféricas
Diabetes mellitus 1 y 2.
Falla renal crónica.
Polineuropatía.
Cirugía de uretra.
Anatómicas o estructurales

Hipospadias, epispadias.
Micropene.
Enfermedad de Peyrone.
Hormonal.
Hipogonadismo
Hiperprolactinaemia.
Hiper e hipotiroidismo.
Hiper e hipocortisolismo (Enf. De Cushing 's)
Medicamentos

Antihipertensivos (diurétios tiazidicos. Etc.)


Antidepresivos.
Anti psicóticos.
Antiandrogenos.
Drogas recreacionales (alcohol, marihuana, cocaína etc.
Psicogénicos

Tabla 1. Causas de disfunción eréctil

10 · Disfunción eréctil
ERRNVPHGLFRVRUJ
Urología 1 10
" El sildenafilo está contraindicado en pacientes que toman nitra-
Ideas clave - tos o fármacos donadores de óxido nítrico, en pacientes con in-
farto agudo de mioca rdio (1 AM) en los últimos seis meses, y en
" La causa más frecuente de disfunci ón eréctil es vascular. pacientes con insuficiencia cardíaca grave o angina inestable.

" La enfermedad endocrina más relacionada con ella es la diabe-


tes mellitus.

Casos clínicos
Paciente de 63 años, en tratamiento a demanda con citrato de 1) Verapamilo.
sildenafilo por presentar disfunción eréctil de años de evolución. 2) Digoxina.
Señale cuál de los siguientes fármacos NO asociaría en ningún 3) lndapamida.
caso a su tratamiento: 4) Mononitrato de isosorbida.

A 78-year-old male goes to your outpatient clinic complaining


Case Study of erectile dysfunction. He has never taken any specific medical
treatment. Mark the incorrect statement:
A 65-year-old patient, with a history of hypertension, goes to
his physician asking for a diagnostic study for his erectile dys- 1) Most patients with erectile dysfunction will be treated with
function. Which of the following options would not be included non-specific treatment or therapies.
in the initial diagnostic workup? 2) The basic diagnostic workup of such a patient should include
the identification of reversible risk factors.
1) Sexual history. 3) Lifestyle changes and controlling risk factors have no role in the
2) Digital rectal examination. current treatment of erectile dysfunction.
3) EKG. 4) The early use of an elevated dose of sildenafil after a radical
4) Measuring his total and free testosterone levels. prostatectomy is associated with preservation of smooth mus-
ele fibers in the human cavernous bodies.

ERRNVPHGLFRVRUJ
Traumatismos del aparato
genitourinario

ENARM
Se han de repasar bien
las Ideas clave. -- 2

3
Descripción
Contusión o hematoma subcapsular no expansivo.
Hematoma perirrenal no expansivo con laceración
cortical a < 1 cm. sin extravasación.
Laceración cortical > 1 cm. Con extravasación urinaria.
Las lesiones del aparato genitourinario son frecuentes en los pacientes Laceración del tejido coricomedular hasta los sistemas
politraumatizados, así que, ante todo paciente con fracturas costales ba- 4 colectores o daño vascular (arteria renal segmentaría o
jas, equimosis o masa en flancos, fracturas de las apófisis transversas, frac- daño a la vena con hematoma contenido).
turas de los cuerpos vertebrales y/o fracturas pélvicas, debe sospecharse 5 Riñan lacerado o daño al pedículo vascular (avulsión).
una lesión de este tipo. Son lesiones más frecuentes en jóvenes y con Tabla 1. Escala de severidad de daño renal
predominio en varones
Otros síntomas y signos son el dolor abdominal o en un fla nco, equi-
mosis en los flancos o cuadrantes superiores del abdomen, masa pal-
11.1. Lesiones del riñón pable secundaria a hematoma retroperitoneal o a urinoma y disten-
sión abdominal.

Son las lesiones más frecuentes del aparato urinario, siendo el mecanis- Las complicaciones inmediatas más frecuentes son: hemorragia, ex-
mo más frecuente (80-85%) el traumatismo contuso directo en el abdo- travasación urinaria que secundariamente da lugar a un absceso y
men, flanco o región dorsal. septicemia.

Clasificación patológica Las complicaciones tardías más importantes son: hipertensión, hidrone-
frosis, fístula arteriovenosa, formación de cálculos, y pielonefritis y la he-
Traumatismo renal menor (85%): engloba los grados I y 11 de la morragia tardía.
Asociación Americana para la Cirugía del Trauma, (AAST). Incluyen la
contusión del parénquima (los más frecuentes), el hematoma cap- En todo paciente que se sospeche de complicaciones (fiebre, dolor in-
sular y las laceraciones corticales superficiales. Rara vez requieren tenso y constante, descenso de hematocrito), debe realizarse un estudio
exploración quirúrgica Tabla 1. de imagen de control.
Traumatismo renal mayor (15%}: grados 111, IV y V de la AAST.
Laceraciones corticomedulares profundas que pueden afectar al Diagnóstico
sistema colector, con extravasación de orina al espacio perirrenal.
Se acompaña a menudo de hematomas retroperitoneales y peri- Para determinar el grado de lesión renal y su función, es fundamental
rrenales. realizar una urografía intravenosa (UIV) que establezca la presencia
Lesión vascular (1 % de los traumatismos contusos): grado V. o ausencia de ambos riñones, defina con claridad los contornos re-
nales, los límites corticales y delimite los sistemas colectores y los
Clínica uréteres. Así como determinar presencia de hematuria microscópica
en pacientes con lesiones donde se sospecha lesión del aparato ge-
La hematuria macroscópica o microscópica después de un traumatismo nitourinario.
indica lesión del aparato urinario, aunque no aparece en todos los casos
(por ejemplo, ante lesiones del pedículo pedículo vascular o avulsión del En el paciente politraumatizado, puede aportar mayor información la rea-
ureter). El grado de hematuria no siempre se corresponde con el grado lización de una TC, siempre y cuando las condiciones hemodinámicas del
de la lesión. paciente lo permitan.

ERRNVPHGLFRVRUJ
Urología 1 11
Si no se determina en su totalidad la extensión de la lesión, puede aso- venosa en busca de la extravasación del medio de contraste en la fase de
ciarse una nefrotomografía o una TC, que además muestra el estado de eliminación.
los órganos vecinos.
Tratamiento
La arteriografía se indica cuando el riñón no se observa bien en la uro-
grafía excretora o cuando se sospecha de lesión vascular y con fines La primera medida es la derivación urinaria mediante nefrostomía. Re-
terapéuticos como la embolización selectiva. quieren tratamiento quirúrgico inmediato.
Lesión del tercio inferior del uréter: el procedimiento de elec-
Las causas más importantes que no permiten la adecuada observación ción es la reimplantación en la vejiga. La ureteroureterostomía
en la TC o UIV son: rotura total del pedículo, trombosis arterial, contu- primaria puede indicarse si hay un corte transversal del uréter. Se
sión intensa que causa espasmo vascular y la ausencia de riñón. usa transureteroureterostomía, si hay urinoma extenso e infec-
ción pélvica.
Los exámenes con isótopos, en la evaluación de urgencia, son menos Lesión del tercio medio y superior: ureteroureterostomía primaria
sensibles que la arteriografía o la TC. o sustitución ureteral (Tabla 2).

Tratamiento Es frecuente dejar un catéter de doble-J transanastomótico, que se retira


después de tres a cuatro semanas de cicatrización, con objeto de conser-
Las medidas terapéuticas iniciales deben dirigirse a la estabilización he- var el uréter en una posición adecuada con un calibre constante, impedir
modinámica y reanimación completa del paciente, con tratamiento del la extravasación urinaria y conservar la desviación urinaria.
shock, control de la hemorragia y la evaluación de las lesiones concu-
rrentes. Pacientes con trauma renal 1-IV con estabilidad hemodinámica
deben ser manejados de forma conservadora, con reposo absoluto y Sitio de lesión Opciones de manejo

monitorización estrecha de signos vitales y niveles de hemoglobina y Uretero - ureteroanastomosis.


Uretero proximal Transuretero - ureteroanastomosis.
hematocrito.
lretero - calycoanastomosis.
Uretero - ureteroanastomosis.
El tratamiento quirúrgico está indicado en: Uretero medio Transuretero - ureteroanastomosis.
Todo paciente inestable (hemorragia retroperitoneal, lesión del pe- Reimplante ureteral.
dículo renal, extravasación urinaria). Reimplante ureteral.
Uretero distal
Los traumatismos renales por penetración (salvo si se ha podido de- Reimplante ureteral con técnica psoas hitch.
terminar el grado de la lesión y resulta ser una lesión menor del pa- Injerto con ileo.
Completo
rénquima sin extravasación urinaria)). Incluso traumatismos grado IV Autotransplante.
si el paciente se encuentra estable pueden ser manejados de forma Tabla 2. Manejo de la lesión ureteral según sitio de lesión
conservadora ..
En el tratamiento de las complicaciones como el urinoma retroperito-
neal o el absceso perirrenal, la hipertensión maligna que requiere re- 11.3. Lesiones de la vejiga
paración vascular o nefrectomía, y en algunos casos de hidronefrosis.

Frecuentemente se presentan debidas a fuerzas externas y asociadas a


11.2. Lesiones del uréter fracturas pélvicas. La lesión iatrógena ocurre en cirugías pélvicas, gineco-
lógicas, herniorrafias y en intervenciones transuretrales.

Son raras, pero pueden ocurrir durante el curso de intervenciones qui- La rotura puede ser:
rúrgicas pélvicas, por heridas de balas, desaceleraciones rápidas en ac- Extraperitoneal: perforaciones por fragmentos de fracturas pélvi-
cidentes, en manipulaciones endoscópicas de cálculos o en resecciones cas. Son las más frecuentes.
transuretrales. lntraperitoneal: golpes directos cuando la vejiga está llena.
Mixta.
La ligadura del uréter conduce a la aparición de hidronefrosis con fie-
bre, dolor en flanco, náuseas, vómitos e íleo, y si es bi lateral, anuria. Clínicamente pueden manifestarse con dificultad para la micción, hematuria
macroscópica, dolor pélvico o en hemiabdomen inferior, abdomen agudo
Si lo que ocurre es extravasación, se forma un urinoma que secunda- (indica rotura intraperitoneal). Como complicación tardía puede aparecer una
riamente provoca estenosis y fibrosis reactiva, junto con hidronefrosis; incontinencia parcial en las lesiones que se extienden hasta el cuello vesical.
si se extravasa hacia la cavidad peritoneal, da lugar a una peritonitis
aguda. Una cistografía demostrará una rotura vesical. La cistoscopia no está indica-
da porque la hemorragia y los coágulos impiden una buena visualización.
En las pruebas de laboratorio, si hay una lesión por traumatismo externo,
aparece hematuria microscópica en el 90% de los casos. Tratamiento
El diagnóstico se realiza mediante una UIV o uretrografía retrógrada. En el Las roturas extraperitoneales se tratan con cateterización uretral perma-
período postoperatorio inmediato, la urografía es el mejor método para nente y cistostomía suprapúbica. Sólo si persiste la extravasación, es ne-
descartar la lesión ureteral o bien en una tomografía con contraste intra- cesario el tratamiento quirúrgico.

ERRNVPHGLFRVRUJ
Manual CTO de Medicina y Cirugía, 3.ª edición

Las roturas intraperitoneales deben repararse por vía transperitoneal (ci- 11.6. Lesiones de. los testículos
rugía reparadora), debiendo dejar cistostomía suprapúbica.

Para definir el daño, se realiza ultrasonografía. Si hay rotura, se trata quirúr-


11.4. Lesiones de la uretra gicamente. En el resto de los casos, el tratamiento es conservador.

A modo de resumen de todo lo anteriormente expuesto, la Figura 1 re-


Son poco frecuentes, más habituales en varones y generalmente aso- coge los procedimientos generales de act uación frente a los traumatis-
ciadas a fracturas pélvicas y a contusiones directas. Se considera ure- mos urológicos.
tra posterior la porción proximal al diafragma urogenital y anterior, la
distal.
Sospecha de traumatismo urológico
Lesiones de la uretra posterior
(prostática y membranosa)
t
Tracto inferior
t
Tracto superior
(uretrorragia) (hematuria)

La uretra membranosa se lesiona con más frecuencia. Los pacientes


t t
Uretrograffa UIVoTC
aquejan dolor abdominal bajo e incapacidad para la micción. El signo
más importante es la presencia de sangre en el meato de la uretra (ure-
trorragia). En un tacto rectal, puede revelarse la presencia de hematoma
~Lesión
Normal
~
Funcional
No funcional
pélvico y desplazamiento de la próstata hacia arriba.
Evaluar
Sondaje vesical Cistotomía Arteriografía
La prueba diagnóstica más importante es la uretrografía. La cateteriza- según grado

ción o ureteroscopia no deben realizarse porque conllevan un alto riesgo t


Sospecha lesión vesical
de producir hematoma e infección y daño más amplio de los desgarros (si hay hematuria)
parciales de la uretra.

Entre las complicaciones destacan estenosis, impotencia e incontinen- Cistografía (+250 mi)

cia.
t t
Rotura
Rotura
Extraperitoneal
Su tratamiento es la cistostomía, seguida de cirugía diferida. lntraperitoneal
(lo más frecuente)

Lesiones de la uretra anterior t


Cirugía Sondaje
(pendular y bulbar) reparadora
Figura 1. Manejo de los traumatismos urológicos

Generalmente hay antecedentes de caída o maniobras con instru-


mentación. Se manifiestan con hemorragia y dolor en el periné, pu-
diendo existir infección por extravasación y estenosis tardía. No debe
intentarse pasar catéter uretral y debe evitarse la micción hasta des-
cartar la existencia de extravasación. Se diagnostican mediante uretro-
grafía retrógrada.

Tratamiento

Si existe laceración, debe realizarse cistostomía suprapúbica.

11.5. Lesiones del pene

Durante las relaciones sexuales, puede ocurrir rotura de la túnica albugí-


nea (fractura de pene). Provoca dolor y hematoma y precisa tratamiento
quirúrgico (Figura 29). Figura 2. Fractura de pene a la exploración física

11 · Traumatismos
del aparato genitourinario
ERRNVPHGLFRVRUJ
Urología 1 11
" Requieren tratamiento quirúrgico los pacientes inestables, los
Ideas clave traumatismos renales por penetración con extravasación urina-
ria, urinomas retroperitoneales o abscesos perirrenales.
" La hematuria macroscópica o microscópica después de un trau-
matismo indica lesión del aparato urinario. " Las lesiones vesicales pueden ser extraperitoneales, que al igual
que las lesiones de uretra, se asocian a fracturas pélvicas, e intra-
" Para determinar el grado de lesión renal y su función, se puede peritoneales, producidas por golpes directos cuando la vejiga
realizar una urografía. En el paciente politraumatizado, puede está llena.
ser más útil la TC.

2) Las secuelas pueden ser graves y deben prevenirse con cirugía


Casos clínicos precoz.
3) En muchos casos se resuelve con compresión de la zona afecta.
Paciente que, tras una caída a horcajadas, refiere sangrado por el 4) Lo más probable es que precise un ingreso hospitalario prolon-
meato uretral con dificultad para la micción. Presenta un hema- gado.
toma perineal importante.

1) Debe realizarse sondaje inmediato para prevenir complicacio- Paciente precipitado desde 4 m de altura mientras limpiaba
nes. unos cristales. Tras valoración y estabilización iniciales, se deci-
2) Habrá que realizar una TAC abdominal para descartar lesiones de su traslado, durante el que el paciente presenta una micción
asociadas, como en cualquier lesión de uretra posterior. hematúrica.
3) Debe realizarse reconstrucción quirúrgica inmediata en caso de
una rotura completa. 1) Se debe probablemente a una contusión vesical y no debe dár-
4) Debe realizarse una uretrografía retrógrada para valorar la gra- sele mayor importancia.
vedad de la lesión. 2) Puede deberse a lesiones a varios niveles del aparato urinario y
una TAC abdominal será útil para identificarla.
3) Esto permite focalizar la lesión, descartando así otras lesiones
Varón de 1S años que presenta mareo y palidez tras la visión de abdominales.
sangre que parece provenir de los genitales. Niega traumatismo 4) No se debe colocar una sonda vesical porque no ha presentado
alguno, aunque admite que se masturbaba cuando sucedió. uretrorragia previa a la micción.

1) Lo más probable es que se trate de una rotura parcial de uretra


anterior.

ERRNVPHGLFRVRUJ
~ - - - -UJDlogía_

Recommended reading 1

An abdominal ultrasound is performed on a 42-year-old patient for


another reason. During the study, an image such as the one shown
below is observed in the kidney. lndicate which do you consider to be
the most probable d iagnosis [Figure 1a]:

1. Hydronephrosis.
2. Hypernephroma.
3. Simple renal cyst.
4. Renal abscess.
s. Angiomyolipoma.

When complementary abdominal examinations are performed, particu-


larly ultrasounds, it is common for renal masses to be found incidentally,
such that, currently, the number of renal carcinomas detected in a casual
manner exceeds the number of symptomatic renal carcinomas. However,
in this case it is not a hypernephroma, but a completely benign condi-
tion: a simple renal cyst (we may observe two of them in the ultrasound Figure 1a.
shown).
of age. In arder to make a definitive diagnosis, the following criteria must
An abdominal ultrasound makes it possible to distinguish a solid mass bemet:
from a cystic mass. This feature is essential for the study of renal masses, Imperceptible wa ll in the ultrasound.
since most solid masses correspond to adenocarcinomas, whereas cystic That the limits thereof are well-defined.
masses are almost always simple rena l cysts. Water density, wit hout interna! echoes.
Presence of posterior reinforcement in t he ultrasound.
A simple rena l cyst is a benign, non-neoplastic mass that contains fluid. In the event that a CT or an intravenous urography is performed,
lt is very frequent, since it affects over half the population over 55 years there should be no evidence of contrast being captured.

ERRNVPHGLFRVRUJ
Recommended reading 2

A 35-year-old man visits the physician because he has palpated a mass


in his left testicle. He has no history of trauma or high-risk sexual rela-
tions. The examination confirms the presence of a mass and the atta-
ched ultrasound is performed. What is the most adequate approach?
[Figure 2a]:

1. FNAC of the mass.


2. Surgical biopsy of the mass.
3. Inguinal orchiectomy, following collection of blood sample to de-
termine AFP and b-hCG.
4. Transscrotal orchiectomy with hemiscrotal resection.
S. Monitoring by ultrasound, since it is a normal testicular image.

Testicular tumours are the most frequent malignant neoplasias in males Figure 2a.
between 20 and 35 years of age, except far leukaemias. The most fre-
quent form of clinical presentation is as a scrotal mass. befare and after the orchiectomy we should measure the levels of hCG and
AFP. High concentrations of hCG and AFP decrease according to lirst-order
A testicular ultrasound is a simple, reliable method used to distinguish bet- kinetics; the half-life of hCG is 24-36 hours and that of AFP is 5-7 days. These
ween salid and cystic masses. lt is also very useful to determine whether two markers should be serially analysed during and after the treatment. lf any
a mass is dependent on the testicle or the testicular annexes, as well as its or both of these markers increase again or do not decrease according to the
intratesticular position. In the ultrasound shown, we observe a very intense expected half-life, we should suspect tumour persistence or recurrence.
destructuration ofthe testicular parenchyma, which has a clearly heteroge-
neous appearance. Below, we show the appearance of a normal testicular Fuente: García Macarrón J. Casos clínicos en imágenes. Madrid. CTO Edi-
ultrasound, such that you may appreciate the differences. torial, 2012.

When, after the clinical examination


and the ultrasound, the diagnosis is
still not clear, a transinguinal surgical
examination should be performed. lf
this examination confirms the presen-
ce of a mass, the testicle should be re-
moved (answer no. 3 correct).

Overall, 70% of testicular tumours ma-


nifest sorne marker. lt is important to
determine these markers, not only far
diagnostic purposes, but primarily with a
view to the subsequent follow-up, since Figura 2b-2c. To the right, normal testicle, the parenchyma whereof is visualised as homogeneous in
an increase in them could warn us early the ultrasound. On the contrary, the left image shows a destructured parenchyma, with heterogeneous
on about a recurrence. Far this reason, echogenicity and sorne calcification, which suggests malignancy.

ERRNVPHGLFRVRUJ
.. -.. -- - _______ __Utolo.gía

Solucionario
Casos clínicos/Clase study

. . .w ..__ '

~
1 3 1
1 3
1 1
2 3 1 2 3
2 3 4
'
1 1 2
1
4 4 1 2 4
5 2 2 1 3 3
4 4 3
3
2 4 1 5 2
1 4
4 1
2 4 1 2 4
3
1 3 3 4
1
1
2 3 i 1 4
5 1
3 2
1

1
i 1 1
4 4 1
1
2 2
4
1 1 1 3 4
2 3 1
4 4
6 1
3 3 1 3
4 1
i 2 3
5
1 4 3 2
7
2 4 4 2
1

8 1 4 6 1 3
10 1 4 1 2 2
4 3
1
11 2 3 ! 4 4
3 2 7 ' 1 3
2 1
3 2
4 4
'
9 1 2
!
10 1 1 3
i
: 2 3
HGLFRVRUJ

11
Uro IOgÍ a I Bibliografía

Bibliografía

¡ji Castiñeiras Fernández J. Libro del Residente de Urología. Gráficas oJ Jiménez Cruz JF, Rioja Sanz LA. Tratado de urología. Prous Science,
Marte, Madrid. 2006. Barcelona. 2006.

1:11 European Association of Urology. Guías clínicas Europeas 201 O. óJ Resel Estévez L, Moreno Sierra J. Tratado de oncología urológica.
EAU, Arnhem. 201 O. Grupo Saned, Madrid, 2003.

LiJ Grupo CTO, Manual CTOde Urología. 9.• ed., CTO Editorial, Madrid, oJ Wein AJ, Kavoussi LR, Novick AC, Partin AW, Peters CA. Campe/1-
2014. Walsh Urología. Panamericana, Argentina. 2008.

ERRNVPHGLFRVRUJ

You might also like